Главная Юзердоски Каталог Трекер NSFW Настройки

Космос и астрономия

Ответить в тред Ответить в тред
Check this out!
<<
Назад | Вниз | Каталог | Обновить | Автообновление | 511 75 145
Тред тупых вопросов №180 Аноним 09/07/23 Вск 09:49:14 757854 1
image.png 1011Кб, 1200x800
1200x800
image.png 1639Кб, 1332x888
1332x888
image.png 2010Кб, 1906x1072
1906x1072
image.png 2483Кб, 1366x768
1366x768
Тред вопросов о жизни, Вселенной и всем таком.
Спрашиваем то, за что в других местах выдают путевку в биореактор. Здесь анонимные учёные мирового уровня критически рассмотрят любые гениальные идеи и нарисованные в Paint схемы.
Прошлый тред >>755890 (OP) распидарасило в гравитационной сингулярности.
Аноним 09/07/23 Вск 10:02:49 757856 2
Если Земля плоская, то и Марс тоже плоский?
Аноним 09/07/23 Вск 10:42:06 757858 3
Аноним 09/07/23 Вск 10:42:11 757859 4
>>757856
Вселенная плоская
Аноним 09/07/23 Вск 10:50:13 757860 5
202108150629016[...].jpg 108Кб, 880x440
880x440
>>757859
Я читал, что наша Вселенная - пончки
Аноним 09/07/23 Вск 10:50:58 757861 6
Почему Семихатов, Штерн, Масленников и Попов (ну и многие другие) так категорично утверждают, что межзвездные перелеты невозможны? Ну они дауны блять, не люди же летать будут, а роботов сделаем.
Аноним 09/07/23 Вск 10:58:24 757862 7
>>757861
>межзвездные перелеты невозможны?
А почему при межзвездных перелетах обязательно лететь в плоскостях солнечных систем?
Почему нельзя вылететь вверх из плоскости и солнечной системы?
Аноним 09/07/23 Вск 11:03:18 757863 8
>>757862
>Почему нельзя вылететь вверх из плоскости и солнечной системы?
И это даст нам ... что?
Аноним 09/07/23 Вск 11:15:46 757864 9
Где найти софт или сайт, где показано движение планет в 3д в режиме реального времени или на конкретную дату. Спасибо.
Аноним 09/07/23 Вск 11:47:47 757865 10
Аноним 09/07/23 Вск 11:54:14 757869 11
>>757865
спасибо БОЛЬШОЕ!
Аноним 09/07/23 Вск 12:16:31 757877 12
Расширение Вселенной это вращение её в пятом измерении. Мнение?
Аноним 09/07/23 Вск 12:20:18 757878 13
forrest-gamp.jpg 126Кб, 1364x578
1364x578
>>757877
Да ты гений, анон, ты уже подал заявку в нобелевский комитет?
Аноним 09/07/23 Вск 12:24:47 757879 14
Аноним 09/07/23 Вск 12:29:06 757880 15
>>757879
Потому что это статья про математику, а не астрофизику.
Аноним 09/07/23 Вск 12:42:23 757883 16
Аноним 09/07/23 Вск 12:43:45 757884 17
Аноним 09/07/23 Вск 12:46:53 757885 18
>>757884
Это фотографии или что это?
Аноним 09/07/23 Вск 12:47:56 757886 19
16196986320740.jpg 61Кб, 755x755
755x755
>>757885
>Это фотографии или что это?
Аноним 09/07/23 Вск 12:49:13 757887 20
>>757885
Выдумка художника на основе научных данных.
Аноним 09/07/23 Вск 13:09:42 757890 21
>>757860
Речь о геометрии, а не о топологии. Разные вещи
>>757861
>Почему
Потому что зашоренные хуесосы. То, что нашу планету активно посещают внеземные цивилизации, является красноречивым доказательством обратного
>>757877
>Мнение?
Хуета и говно
Аноним 09/07/23 Вск 13:16:42 757891 22
>>757860
Погоди, Перельман же доказал гипотезу, по которой это именно сфера. Стянуть петлю из ниток с мяча, вот это вот.
Аноним 09/07/23 Вск 13:19:56 757892 23
m60.jpg 7551Кб, 6352x4312
6352x4312
Почему в центре Млечного пути такая нищая черная дыра массой всего 4 миллиона солнечных? Вон М60 по размерам такая же, а ЧД здоровенная 4,5 миллиарда солнц.
Аноним 09/07/23 Вск 13:50:24 757895 24
image.png 665Кб, 1526x1428
1526x1428
Когда мы с помощью ветряков получаем энергию от ветров, мы же потихоньку отбираем энергию у атмосферы? Если мы ветряками всю энергию атмосферы отберём, то получается на Земле больше не будет ветров? Зеленые, вы этого хотите?
Аноним 09/07/23 Вск 13:52:47 757896 25
>>757895

так ветра рождаются от разности температур, это возобновляемый ресурс.
Аноним 09/07/23 Вск 15:25:22 757905 26
>>757895
Дон Кихот, перелогинься.
Аноним 09/07/23 Вск 15:34:54 757906 27
>>757892
М60 это галактика маминой подруги, очевидно же.

Судя по виду галактики она явно старше МЛ и тамошня дыра насосала больше вещества.
Аноним 09/07/23 Вск 15:45:04 757907 28
>>757906
>явно старше МЛ
Возраст молока 13,6 млрд лет, куда уж старше то?
Аноним 09/07/23 Вск 16:00:14 757912 29
>>757907
>13,6 млрд лет
13 млрд
Аноним 09/07/23 Вск 16:02:43 757913 30
>>757912
ты чо чмонька тя в гугле забанили шоль уахахаах хд хд хд х д)))))🤣🤣🤣🤣🤣🤣
Аноним 09/07/23 Вск 16:10:16 757914 31
>>757913
Ладно, сын шлюхи, ты прав. 13,6 млрд, правда. Но ты все равно соси залупу тухлую
Аноним 09/07/23 Вск 16:48:49 757921 32
>>757895
>
>Когда мы с помощью ветряков получаем энергию от ветров, мы же потихоньку отбираем энергию у атмосферы?
Вот только ветра берутся изначально за счёт энергии солнечного света на атмосферу, землю и воду
Аноним 09/07/23 Вск 17:27:31 757924 33
>>757896
Хорошо - приливные электростанции тоже возобновляемый ресурс?
Аноним 09/07/23 Вск 17:28:34 757925 34
Какого цвета Солнце внутри нашей полой Земли?
Красное?
Красный карлик?
Получается там всё залито красный цветом?
Демоническое зрелище?
Аноним 09/07/23 Вск 19:20:05 757929 35
Почему интерес любительский очевидно к космологии не соседствует с интересом к космическим полетам?
Я вот люблю смотреть/читать про историю Вселенной, формирование галактик, разные эффекты умозрительные.
Но при этом вообще плевать на полеты, запуски, Маска, старшипы и пр. Это все кажется мне как-то мелко, глупо и будто не стоит внимания. Так и не смог заставить себя заинтересоваться.
Я типа дурачок гуманитарий?
Аноним 09/07/23 Вск 19:24:20 757930 36
Эй, учёные, постоянно вижу при указании расстояний то световой год, то какие-то парсеки. Заебали, выберите себе уже одну стандартную единицу. Как футы и метры какие-то, ну в самом деле.
Аноним 09/07/23 Вск 19:59:44 757935 37
>>757861
Штерн как раз активно за топит.
Аноним 09/07/23 Вск 20:10:40 757936 38
>>757925
Внутри земли не солнце, а черная дыра с аккреционным диском.
Аноним 09/07/23 Вск 20:24:53 757937 39
Аноним 09/07/23 Вск 20:31:59 757938 40
>>757936
Почему тогда подземные базы аннунаков не затягивает?
Аноним 09/07/23 Вск 20:58:37 757941 41
>>757938
>Почему тогда подземные базы аннунаков не затягивает?
Ты идиот? Потому что они выше горизонта событий
Ну ты и жирдяй, тебе не не место в космаче, вали траллить в бэ
Аноним 09/07/23 Вск 22:16:39 757944 42
>>757925
Земля плоская, очевидно же.
Аноним 09/07/23 Вск 22:20:58 757945 43
>>757944
Как ты внутрь плоской Земли засунешь Солнце? Совсем дурачок, да?))
Аноним 09/07/23 Вск 22:22:03 757946 44
>>757930
а. е. - стандартная единица измерения расстояний внутри звёздных систем.
световой год - для ближайших звёзд.
парсек - для галактик
Три стандартных единицы. Что тебя не устраивает?
Аноним 09/07/23 Вск 22:22:48 757947 45
>>757945
В этом и прикол. Нет там никакого солнца.
Аноним 09/07/23 Вск 22:24:32 757948 46
>>757944
Ты мне эту херню не загоняй
Ведь чтобы скрыть правду - нужно её растворить в море лжи
Поэтому были выпущены псы с теориями плоской Земли чтобы в этом утопичном хаосе растворить истину о полой Земле
Типо нет никакой разницы между полой и плоской - "все они несут чушь, буду верить книжкам от рептилоидов с металлическим ядром" - так рассуждает среднестатистический Вася
А у рептилоидов внутри полой Земли база, они и рады радешеньки, что тупоголовые Васьки даже в уме не допускают возможность существования полой Земли
Аноним 09/07/23 Вск 22:26:27 757949 47
>>757947
>Нет там никакого солнца.
То что внутри полой Земли есть Солнце - доказывается наличием общепланетарного магнитного поля
Аноним 09/07/23 Вск 23:13:40 757955 48
>>757861
На текущих технологиях полет людей таки невозможен.
Но это не значит, что в будущем не будет остановлено биологическое старение, излечены болезни, изобретён анабиоз или вообще сознание не будет перенесено в кудахтер.

Мне больше интересно, почему Сурдин такой долбоеб? Он на серьёзных щах уверен в следующих вещах:
- сигнал от инопланетян будет обнаружен через 8 лет
- реальность проекта brakethrough starshot
- на Луне под грунтом есть бактерии
Аноним 09/07/23 Вск 23:16:10 757956 49
Аноним 09/07/23 Вск 23:21:25 757957 50
>>757924
да, приливы за счет воздействия гравитации луны существуют
Аноним 10/07/23 Пнд 00:51:13 757960 51
Почему в гамма-излучении частицы летят с высокой энергией и они опасны, а в радиодиапазоне нет. Объясните их природу.
Аноним 10/07/23 Пнд 01:37:01 757964 52
>>757960
Свч вполне могут зажарить типа мощных радаров, думаю и другие волны если очень мощные не особо полезны, даже просто магнитное поле типа как у нейтронных звезд может повырывать у тебя электроны, ну или просто захуевеет как под ЛЭП. Гамма это просто фотоны высоких энергий, они ионизируют, есть ещё уф, особенно жёсткий, он тоже обжигает так нехило, в лампах ртутных без люминофора он именно и лучит.
Аноним 10/07/23 Пнд 08:02:05 757978 53
>>757929
>Это все кажется мне как-то мелко, глупо и будто не стоит внимания
Это так и есть. Это все бизнес и не более
>>757930
Понятие "световой год" используют, когда пытаются что-то пояснить неспециалистам - это более простой термин, чем "парсек". Профессиональные астрофизики пользуются именно парсеками, килопарсеками, мегапарсеками и гигапарсеками
>>757960
У гамма-квантов охуительная энергия - они выбивают электроны из атомов и ионизируют вещество. Ионизация повреждает ДНК и приводит к активизации мутагенеза, который, в свою очередь, может привести к раку. Поэтому гамма-излучение такое опасное
Аноним 10/07/23 Пнд 08:55:40 757979 54
>>757978
>Ионизация повреждает ДНК
Почему?
Аноним 10/07/23 Пнд 10:35:55 757988 55
>>757979
ДНК это большая линейная молекула, ионизация одно из звенья цепи разрушает связи и молекула разрушается на части.

Но это все хуйита.
Главная мишень для гамма квантов это молекулы воды и тяжелые ионы(кальция/фосфаты/железа). Ионизация превращает воду в злые радикалы, которые мигом взаимодействуют с органическими молекулами, иногда разрывая их на части. ДНК особенно уязвима к этому.
Тяжелые ионы так вообще брутальный пиздец. Они могут на себя наматывать молекулы и рвать в нескольких местах, поэтому в клетке они часто находятся в составе охлаждающих их траханье других молекул. ионизация выбивает тяжелый ион из него и она начинает беспредел.
Аноним 10/07/23 Пнд 11:32:01 757993 56
>>757988
Тяжелые ионы - это просто с огромной скоростью летящие протоны?
Аноним 10/07/23 Пнд 12:39:05 757998 57
image.png 1266Кб, 1500x843
1500x843
Представьте куриное яйцо.
Обычное варёное куриное яйцо в разрезе.
Каково оно? Если визуально разделить яйцо, то - если утрировать - яйцо имеет форму капли и его можно разделить на округлую часть и на конус
Желток чаще всего расположен более-менее в центре или в округлой части.
А теперь представьте себе, что у нас желток оказался в конусе яйца, полностью покинув округлую половину. Такое бывает, хотя и не часто.
А теперь представим такое же, но для планеты. Желток - ядро, белок - мантия и кора. Само собой центр масс в желтке и центр гравитации тоже. И как тогда будут проходить планетарные процессы при таких условиях? Если жители такого яйца запустят спутники на орбиты по экватору, то они будут видеть странную неровность полушарий, когда округлость существенного объёмнее конуса? А как будет происходить запуск спутника по полярной орбите?
Не будет ли каких-нибудь аномалий в связи с такой планетарной особенностью?
Аноним 10/07/23 Пнд 12:54:05 758000 58
>>757993
Нет, это атомы с электрическим зарядом, который у них появляется после выбивания электрона гамма-квантом
Аноним 10/07/23 Пнд 15:13:52 758007 59
image.png 382Кб, 585x923
585x923
сидите тут как фуфелы и новостей не знаете
Аноним 10/07/23 Пнд 15:34:24 758011 60
>>757885
Фотороботы по сути своей. Художник рисует объект со слов наблюдателя объекта.
Аноним 10/07/23 Пнд 15:40:08 758012 61
Аноним 10/07/23 Пнд 16:01:00 758013 62
Аноним 10/07/23 Пнд 17:09:54 758021 63
>>758013
А на оф сайте наса где эта новость?
Аноним 10/07/23 Пнд 17:10:24 758022 64
>>758013
Беспезды, кстати.

https://vista.news/science/kosmos/303141

Cвидетельство о регистрации СМИ ЭЛ № ФС 77 - 68546. Сетевое издание "VistaNews" зарегистрировано в Федеральной службе по надзору в сфере связи, информационных технологий и массовых коммуникаций (Роскомнадзор) 31 января 2017г.
Аноним 10/07/23 Пнд 17:13:18 758023 65
>>758013
В Совке ничего нельзя было протащить в печать без цензуры, но при этом ухитрялись печатать про марсиан и прочую западную желтуху.
Аноним 10/07/23 Пнд 18:06:42 758028 66
a.jpg 391Кб, 950x669
950x669
>>757964
>>757978
А почему ультрафиолет опасен? Он жеж на границе видимого света. Как он воздействует на глаза и организм??
Аноним 10/07/23 Пнд 18:13:32 758030 67
>>758023
>запад совкам говна в жопу залил даже несмотря на стоящую на страже трезвомыслия совковую цензуру
Аноним 10/07/23 Пнд 19:03:14 758031 68
>>758028
Точно так же. Ультрафиолетовые фотоны обладают достаточной энергией, чтобы ионизовать вещество
Аноним 10/07/23 Пнд 20:20:23 758036 69
16834177571261.gif 4802Кб, 640x348
640x348
>>758007
>>758012
>>758013
>>758023
>>758030
Вы чё за ёбаный бред несёте?!
Какие нахуй совки, конспирологи и найденный засыпанные пеплом тела на Марсе?!!
Аноним 10/07/23 Пнд 21:42:45 758042 70
>>757862
>Почему нельзя
Потому что нет на перпендикулярных орбитах удобных рогаток типа наших газовых гигантов. А так-то теоретически можно, но для этого дельты нужно пиздец.
Аноним 10/07/23 Пнд 23:36:50 758048 71
>>757949
Откуда у Солнца магнитное поле?
Аноним 10/07/23 Пнд 23:39:30 758049 72
>>758048
Конвенция в плазме.
Аноним 10/07/23 Пнд 23:55:44 758050 73
>>758028
Ну как загар, жёсткий отсекается атмосферой, но в космосе и под лампами сильно обжигает, а все мелкое моментально стерилизует. Почему так, ну наверное из-за длинны волны.
Аноним 11/07/23 Втр 00:04:18 758051 74
>>757998
Все массивные планеты проходят стадию дифференциации, все тяжёлое уходит в ядро, лёгкое - на поверхность. Всякие мелкие планеты вполне могут быть кривыми косыми, но они и атмосферу не удержат. А вообще скорее есть мелкие планеты ядра бывших планет, с которых от удара сдуло все лёгкое, как Меркурий или Психея. А так будет смещенный центр масс, вращение наверное рывками, кривое магнитное поле, если вообще оно будет, гравитация будет тоже неравномерна по поверхности.
Аноним 11/07/23 Втр 00:09:01 758052 75
>>758049
Подожди, а внутри Солнце тоже полое?
Аноним 11/07/23 Втр 11:14:31 758091 76
>>758052
Нет же, если бы оно было полое, то конвекция не самозапускалась.
Аноним 11/07/23 Втр 11:15:10 758093 77
>>758052
Плоское. Это тупо плоский светильник приколоченный к небесной тверди.
Аноним 11/07/23 Втр 18:52:10 758137 78
Может ли озон после грозы или дождя сильно отравить человека?
Аноним 11/07/23 Втр 19:58:42 758145 79
>>758137
Нет. Больше вреда будет приносить оксиды азота, которых производиться гораздо больше озона при электрических разрядах.
Аноним 12/07/23 Срд 00:05:31 758191 80
>>758051
>Все массивные планеты проходят стадию дифференциации, все тяжёлое уходит в ядро, лёгкое - на поверхность. Всякие мелкие планеты вполне могут быть кривыми косыми, но они и атмосферу не удержат. А вообще скорее есть мелкие планеты ядра бывших планет, с которых от удара сдуло все лёгкое, как Меркурий или Психея.
Но вопрос не об этом. Если я вопрошаю, как оно вообще на Титане, то зачем отвечать А ВОТ НА ГАНИМЕДЕ...
Аноним 12/07/23 Срд 10:22:24 758269 81
>>758137
Для серьезного воздействия необходима определенная концентрация на м³ воздуха. Раз в 10-30 большая чем та, при которой ты начинаешь чувствовать запах озона во время грозы. При естественных условиях, это крайне маловероятно.
Куда более опасно безответственное использование озонаторов разных сортов, собственно основное назначение которых стерилизация предметов или помещений.
Аноним 12/07/23 Срд 11:13:22 758274 82
Космач,вот я регулярно слышу от всяких сурдиных, что сейчас ежедневно открывают новые объекты Солнечной Системы -всякие там мелкие астероиды.А где можно отслеживать регистрацию этих новых объектов, какой-нибудь онлайн каталог или типа того?
Аноним 12/07/23 Срд 14:14:54 758294 83
Аноним 12/07/23 Срд 14:48:51 758301 84
>>758294
Cпасибо! Меня интересует именно ежедневная информация об открытых за прошедшие сутки телах.Кто-нибудь знает подобные?
Аноним 12/07/23 Срд 19:41:03 758347 85
image 48Кб, 823x490
823x490
>>757854 (OP)
возможно ли такая конфигурация где бы атмосферы двух планет были связаны?
Аноним 12/07/23 Срд 19:45:33 758348 86
Возможна ли кипящая планета? Например, планета-океан из воды в приливном захвате.
На дневной стороне температура у поверхности 120°. Вода из океана постоянно кипит и испаряется. Пар поднимается вверх и устремляется на ночную сторону, где постоянно идёт дождь.
Аноним 12/07/23 Срд 19:48:38 758349 87
>>758347
Нет.
Обе планеты будут друг другу приливом разрывать и такая система не устойчива - будет перетекание вещества(в том числе силикатной мантии) с одной планеты на другую, атмосферу скорее всего выбросить нахуй.
Аноним 12/07/23 Срд 19:51:21 758351 88
>>758348
Ио нахуй. Хотя это спутник, но там вода прям кипит аж в космос.
Внезапно Венера, только кипит там не вода, а углекислый газ.
Аноним 12/07/23 Срд 19:56:18 758352 89
0c4257b2e1d2e4e[...].jpg 126Кб, 736x1375
736x1375
Астероиды же имеют свое притяжение, чтобы по ним можно было ходить? Если да, то можно ли представить себе такой астероид, по которому можно ходить, но, при желании, можно просто выпрыгнуть на своих двоих в открытый космос? Вот это я херню сморозил, да.
Аноним 12/07/23 Срд 20:04:44 758353 90
>>758348
Полнота ких наокрывали -горячие юпитеры, горчие нептуны и проч. Есть даже такие, где камень и метал кипят, "планеты каменных дождей".
Аноним 12/07/23 Срд 20:07:06 758356 91
>>758353
Хуёво на такой планете жить...
Аноним 12/07/23 Срд 22:10:12 758397 92
Как установили что реликтовое излучение это эхо бы,а не просто шум вселенной?
Аноним 12/07/23 Срд 22:27:59 758398 93
>>758349
>такая система не устойчива
Это в первое время. Потом сбалансируется))
Аноним 12/07/23 Срд 22:31:10 758399 94
>>758352
Есть асероиды с очень слабой силой притяжения, но встать на них своими двумя вылезя и звездолёта и ходить по ним будет проблемно - при попытки это сделать ты просто оттолкнёшься от подобной каменюки как воздушный шарик летая отталкивается от пола, стен и других предметтов.Когда брали пробы грунта с астероидов,то подгоняли к ним космический аппарат и отсасывали с них пыль специальным пылесосом, потому что даже просто потрогав поверхность щупом каменюку просто бы отолкнулив сторону, не говоря уже о посадке.
Аноним 12/07/23 Срд 23:05:19 758401 95
>>757856
Марс это планида, а земля это твердь.
Аноним 12/07/23 Срд 23:08:48 758402 96
>>758401
Марс - это шоколадный батончик, дебс!
Аноним 12/07/23 Срд 23:13:58 758403 97
>>758352
Крупные астероиды вполне обладают достаточным притяжением, чтобы от одного пинка не улететь навсегда. На таких вполне можно ходить, но с трудом. Однако таких астероидов около сотни.
Большинство астероидов довольно малые и вторая космическая скорость там считанные метры в секундах или даже меньше. Сделать шаг на них уже не получиться.
Аноним 12/07/23 Срд 23:33:09 758404 98
>>758402
А как этот батончик капитал написал по-твоему?
Аноним 12/07/23 Срд 23:54:05 758405 99
DVZQ796XUAU-mJw.jpg 53Кб, 800x610
800x610
>>758347
Конечно, в лексе такое было планета вода и планета огонь, они на дирижаблях переворачивались и так попадали с одной на другую.
Аноним 13/07/23 Чтв 08:28:44 758419 100
>>758397
>а не просто шум вселенной?
Что за "шум Вселенной" такой в виде микроволнового излучения, заполняющего все пространство? Кто нашумел-то? Господь?
Теория горячей Вселенной предсказывала, что все пространство должно быть заполнено микроволновыми фотонами, образовавшимися спустя 400 тыс. лет после начала расширения - тогда, когда произошла рекомбинация, т.е. объединение электронов с ядрами и образование нейтральных атомов. Пензиас и Вилсон в 60-х и обнаружили эти фотоны - в точности так, как предсказывала теория. Когда предсказание теории полностью подтверждается, значит, это верная теория. Поэтому космический микроволновый фон - это действительно эхо Большого взрыва, а не магический "шум Вселенной"
Аноним 13/07/23 Чтв 08:42:33 758420 101
>>758419
Ты не ответил на вопрос,как они определили,что это именно оно?
Аноним 13/07/23 Чтв 08:47:15 758421 102
>>758420
Теория предсказала эффект - эффект найден. Так и определили
Аноним 13/07/23 Чтв 08:51:39 758422 103
>>758421
Охренеть ответил,наверно и учёным надо так поверить,они сказали шо это оно,значит оно
Аноним 13/07/23 Чтв 09:01:28 758423 104
>>758422
Ты в курсе, как работает наука? Не "ученые сказали", а теория дала конкретные предсказания - эти предсказания полностью подтвердились наблюдательными данными. Значит, данная теория адекватно описывает реальность. Как раз-таки слова в науке не значат нихуя.
Если ты сейчас построишь модель, которая будет предсказывать конкретные наблюдаемые эффекты, свидетельствующие, что CMB - это не эхо Большого взрыва, а какой-то волшебный "шум Вселенной", и если эти эффекты будут обнаружены, то нынешние космологические модели выкинутся на помойку; микроволновый фон будет обосновано интерпретироваться не как эхо Большого взрыва, а как некий "шум Вселенной". Если у тебя нет гипотезы с конкретными предсказаниями, тогда и говорить не о чем. Без гипотезы и предсказаний все твои предположения - пердеж в лужу
Аноним 13/07/23 Чтв 09:53:46 758424 105
>>758420
>>758422
Объясни с теоретическим обоснованием, что такое "щюм всиленай", пидор!
Аноним 13/07/23 Чтв 15:20:13 758461 106
Постоянно читаю, что основной способ обнаружения экзопланет — это их транзит.
Я не пойму, ведь чтобы увидеть транзит планеты через звезду, надо находиться в плоскости орбиты планеты. Галактика же не плоская. Вот есть заезда в 15 св.г от нас и сдвинута ещё по направлению оси Солнца на 1 св.г например. Мы же не тогда не увидим никакого транзита?
Тогда почему это основной метод?
Аноним 13/07/23 Чтв 16:00:56 758462 107
Аноним 13/07/23 Чтв 16:11:46 758463 108
>>758461
Да, транзит работает, только если плоскости эклиптики более-менее совпадают.

Ещё есть метод угловых скоростей, фиксируется движение звёзды вокруг общего с планетой центра масс.
Аноним 13/07/23 Чтв 16:16:38 758464 109
1.png 4Кб, 673x520
673x520
>>758461
Если планетная система расположена плашмя по отношению к нам,то мы смотрим как звёздочка регулярно подёргивается в разнне стороны - это значит,то на неё влияет обращающаяся вокргу неё плонета.
Аноним 13/07/23 Чтв 20:16:11 758480 110
Какую максимальную гравитацию выдерживают живые существа? Должны же были быть эксперименты, где мышей, кузнечиков и микробов раскручивали в центрифуге.
Аноним 13/07/23 Чтв 20:24:19 758483 111
image.png 1067Кб, 985x922
985x922
Поясните за газо-пылевые облака в областях активного звездообразования по типу пикрила.
Какая там плотность вещества?
Что-то на уровне верхних слоев атмосферы или сильно разреженнее?
Аноним 13/07/23 Чтв 21:08:02 758494 112
>>758480
>Должны же были быть эксперименты, где мышей, кузнечиков и микробов раскручивали в центрифуге.
Даже людей раскручивали в середине прошлого века в некоторых странах.
Аноним 13/07/23 Чтв 21:32:04 758503 113
>>758480
Зависит от времени. Доли секунды человек может выдержать десятки g, минуты можно выдержать до 10 g, 1-3 можно терпеть часами, но это явно не полезно для организма.
Аноним 13/07/23 Чтв 22:35:27 758513 114
>>758461
>Тогда почему это основной метод?
Потому что простой и не требует каких нибудь йоба 30 метровых зеркал для телескопа. Но ситуативный, да.
А вот если не поскупиться и построить йобу а лучше ещё и вне атмосферы, то можно и прямыми наблюдениями заниматься. Короче все упирается в инструменты, как и всегда.
Аноним 14/07/23 Птн 01:20:00 758539 115
>>758513
>А вот если не поскупиться и построить йобу а лучше ещё и вне атмосферы, то можно и прямыми наблюдениями заниматься.
Для прямых наблюдений экзопланет у йоба телескопа ещё должен быть коронограф
Аноним 14/07/23 Птн 01:42:29 758541 116
image.png 1138Кб, 1800x900
1800x900
Вопрос такой: если Солнечная Система сформирована из облака, тогда почему Солнце оказалось состоящее полностью из легкого материала (Водород, Гелий) и Кислорода там меньше 1%, тогда как все остальное имеет состав вообще другой - например, дохуя воды даже на расстоянии 1 светового года? Почему более тяжелую хуйню выбросило подальше?
Аноним 14/07/23 Птн 02:39:58 758543 117
1689291596346.jpg 438Кб, 1080x1152
1080x1152
>>758480
НИХУЯ СЕБЕ
400 тысяч жэ.
Да блять при столкновении метеорита нагрузки меньше.
Аноним 14/07/23 Птн 02:53:40 758544 118
image.png 73Кб, 982x266
982x266
>>758543
Ну грибы могут рассеивать споры при 180 000.
Аноним 14/07/23 Птн 08:41:14 758557 119
>>758541
Во Вселенной вообще ~ 75% водорода и 23% гелия( если говорить о "нормальном" барионном веществе, а всяких тёмных материях-энергиях). В абсолютных числах и кислорода, и железа,и золота и чегоугодно в Солнце больше чем в Земле.
Аноним 14/07/23 Птн 08:53:11 758558 120
Япония во время испытаний взорвался ракетный двигатель

На испытательном полигоне Японского агентства аэрокосмических исследований (JAXA) во время тестового полета взорвался ракетный двигатель Epsilon S. На кадрах видно, что взрыв произошел примерно через минуту после запуска двигателя второй ступени.

Это уже не первая неудача японской космической программы. Ранее основная японская ракета-носитель H3 средней грузоподъемности самоликвидировалась в марте, так как произошел отказ двигателей второй ступени. В октябре отказала уже твердотопливная ракета Epsilon-6.
Аноним 14/07/23 Птн 10:11:45 758561 121
Если вселенная расширяется,то на каком расстоянии от нашей галактики находились очень дальние галактик в момент образования?
Аноним 14/07/23 Птн 10:49:27 758565 122
>>758561
Мочоные не знают, они там костыль придумали что якобы был период резкого расширения а потом он типа замедлился, как на самом деле хуй знает. Щас вообще на веббе и влт регистрируют такие галактики с такими параметрами, которые по идее не могли образоваться буквально за пол лярда лет. Значит что-то не так скорее всего, либо вселенная намного древнее, либо ещё что-то.
Аноним 14/07/23 Птн 10:56:08 758566 123
>>758561
>в момент образования?
В момент образования чего? Если в момент образования непосредственно галактик, то можешь сам посчитать: берешь современное значение постоянной Хаббла, возраст галактики и пересчитываешь эти параметры между собой.
Если речь идет про Вселенную, то ни на каком: когда Вселенная родилась, в ней не было вещества. Оно появилось после стадии инфляции, т.е. через 10^-35 сек после рождения Вселенной
Аноним 14/07/23 Птн 10:59:18 758567 124
Аноним 14/07/23 Птн 12:14:56 758577 125
>>758566
В момент образования нашей и других галактик в одно время
Аноним 14/07/23 Птн 12:28:16 758580 126
Аноним 14/07/23 Птн 12:36:47 758583 127
Почему у красных карликов такая высокая металличность?
Аноним 14/07/23 Птн 13:27:46 758587 128
>>758577
Ну тогда возьми конкретную галактику с конкретным возрастом и посчитай исходя из современного значения постоянной Хаббла
Аноним 14/07/23 Птн 14:24:16 758589 129
>>758577
А все галактики в одно и то же время образовались?
Аноним 14/07/23 Птн 15:15:54 758594 130
>>758583
Металлы сильно снижают массу джинса и из газа можно самогравитцией собрать более мелкие объекты. Из газа только из водорода и гелия весьма сложно сделать звезду даже нескольких масс солнца.
Аноним 14/07/23 Птн 16:14:15 758597 131
>>758589
Конечно нет. Но галактик очень много, поэтому найти галактику, которая образовалась в то же время, что и Млечный Путь, совсем не сложно
Аноним 14/07/23 Птн 16:39:28 758600 132
image.png 138Кб, 1280x1046
1280x1046
Смотрю на график и вдруг родился вопрос: а существует тройная точка, например, газа неона? А тройная точка серы? А тройная точка железа но ведь железо не возгоняется...?
И что это за миры такие, где такое возможно?
Аноним 14/07/23 Птн 23:55:38 758632 133
image.png 144Кб, 2560x2276
2560x2276
>>758600
Есть для всего кроме Гелия. Железо - все нормально, в ядре Земли как раз такие условия.
Аноним 14/07/23 Птн 23:57:12 758633 134
>>758600
Возгоняется в космосе.

>В условиях лунного «вакуума» уже при 700 - 900 °С еще твердое железо будет испаряться, возгоняться, переходя сразу из твердого состояния в газообразное.
Аноним 15/07/23 Суб 15:21:32 758663 135
Поясните за гравитационный маневр. При нем тело ускоряется, а для ускорения нужна энергия, но гравитация же не отдает энергию. Я вроде понимаю, что это похоже, как ехать с горки. Но вряд ли, если даже в вакууме съехать с горки, можно на такую же заехать и потом еще выше ехать. Так откуда берется энергия для ускорения при гравитационном маневре?
Аноним 15/07/23 Суб 16:04:24 758664 136
>>758663
> Так откуда берется энергия для ускорения при гравитационном маневре?
Используется кинетическая энергия более массивного тела. Грубо говоря, аппарат "ворует" энергию у планеты
Аноним 15/07/23 Суб 16:07:12 758666 137
Swingbyaccanim.gif 145Кб, 240x180
240x180
>>758663
При гравитационном манёвре космический аппарат заимсвует часть кинетической энергии у самой планеты, мимо которой пролетает. Планета своей гравитацией как бы подталкивает КА в направлении своего движения по орбите.
Аноним 15/07/23 Суб 16:31:15 758668 138
>>758664
>>758666
Условная сверхцивилизация использует некое тело для гравитационных маневров миллионы лет. Корабли у них большие и летают они каждую секунду. Как это отразится на этом теле. Оно будет терять массу или что?
Аноним 15/07/23 Суб 17:30:19 758673 139
>>758668
>Условная сверхцивилизация использует некое тело для гравитационных маневров миллионы лет. Корабли у них большие и летают они каждую секунду. Как это отразится на этом теле. Оно будет терять массу или что?
Оно будет терять кинетическую энергию и радиус орбиты. Условно, Юпитер станет поближе к Солнцу.
Аноним 15/07/23 Суб 17:39:09 758674 140
>>758673
А если мы используем центр масс, звезду или черную дыру в центре галактики?
Аноним 15/07/23 Суб 21:00:28 758687 141
пока полчаса пиздели, ракета всё пердела драгоценным горючим или чем? это хладагент, который всё время подливают или он вообще не из ракеты дымит?
https://www.youtube.com/watch?v=dhTvadtW2dc
Аноним 15/07/23 Суб 21:12:41 758689 142
Есть видео моделирование ситуации как человек (предположим бессмертный) погружается в атмосферу юпитера, его поверхность и проч..? Может даже игра симулятор/?
Аноним 15/07/23 Суб 23:13:17 758695 143
>>758689
А прочность у него крепче алмаза? Потому что там температура и давление такие, что даже алмазы потом в жидкость превращаются.
Аноним 16/07/23 Вск 05:41:22 758704 144
>>758668
Орбитальная скорость тела уменьшится -> орбитальный период увеличится, тело отдалится от гравитационного центра.

>>758673
> Условно, Юпитер станет поближе к Солнцу.
Наоборот. Для того чтобы орбита стала по "меньше" скорость должна вырасти т.е. должно быть приращение кинетической энергии.
Для наглядности, орбитальная скорость Меркурия 48 км/сек, Нептуна 5,4 км/сек. Орбитальный период Меркурия 88 дней, Нептуна 165 лет.

>>758674
> А если мы используем центр масс, звезду или черную дыру в центре галактики?
Так не получится. Находясь на орбите вокруг условного Солнца, ты хоть обманеврируйся, дополнительно скорость относительно самого Солнца не изменится.
Аноним 16/07/23 Вск 06:11:14 758707 145
>>758704
> Наоборот. Для того чтобы орбита стала по "меньше" скорость должна вырасти т.е. должно быть приращение кинетической энергии.
Точно? По идее чем выше скорость, тем выше (больше) наша орбита. Набрав очередную космическую скорость, мы вообще "оторвёмся" и улетим.
> Так не получится.
Как тогда аноны предлагают лететь к Альфе Центавра и тормозить около неё гравитационным маневром?
Аноним 16/07/23 Вск 08:17:47 758712 146
Краткая история времени ещё актуальна?
А то постоянно появляются какие-то новости про новые открытия, новые теории. Там ещё ничего не устарело, можно читать?
Аноним 16/07/23 Вск 08:54:46 758713 147
image.png 464Кб, 604x461
604x461
image.png 989Кб, 1024x767
1024x767
Почему с высоты полёт орбитальной станции человек, находящийся на орбитальной станции, не может увидеть оба полюса? Разве высота в сотню километров не достаточна?
Аноним 16/07/23 Вск 09:29:57 758715 148
image.png 5Кб, 528x459
528x459
>>758713
Анон, с гевометрией ты совсем не в ладах? Диаметр земляшки почти 13000 км, 100 км ваще роли не сыграют
Аноним 16/07/23 Вск 11:02:14 758717 149
>>758715
С геометрией Лобачевского?
Аноним 16/07/23 Вск 11:02:37 758718 150
>>758712
Как книжка по истории науки и про русских евреев норм.
А так у него там возраст Вселенной нифига не 13,799, например.
Аноним 16/07/23 Вск 12:11:40 758728 151
Почему всякие дальние кометы,просто крупные астероиды у которых оборот по неск тыс лет не улетают в свободное путешествие,а все равно возвр к солнечной системе?
Аноним 16/07/23 Вск 12:44:59 758732 152
>>758704
>Так не получится. Находясь на орбите вокруг условного Солнца, ты хоть обманеврируйся, дополнительно скорость относительно самого Солнца не изменится.
А если мы используем звезду соседней солнечной системы или дыру соседней галактики?
Аноним 16/07/23 Вск 12:50:31 758733 153
>>758728
С чего ты взял, что не улетают? Если удачно мимо Юпитера пролететь, то он запульнёт к едрене фене.
Аноним 16/07/23 Вск 13:38:42 758734 154
>>758733
А кто то нет,та жа пресловутая 9 планета с якобы периодом в 10к лет
Аноним 16/07/23 Вск 14:02:02 758735 155
>>758734
Её солнце зашвыривает
Аноним 16/07/23 Вск 14:22:13 758736 156
>>758704
>Орбитальная скорость тела уменьшится -> орбитальный период увеличится, тело отдалится от гравитационного центра.
Садись, два.
>Для того чтобы орбита стала по "меньше" скорость должна вырасти т.е. должно быть приращение кинетической энергии.
Для наглядности, орбитальная скорость Меркурия 48 км/сек, Нептуна 5,4 км/сек. Орбитальный период Меркурия 88 дней, Нептуна 165 лет.
Хуюшки. Ты не учитываешь потенциальную энергию и некорректно понимаешь третий закон Кепплера. Квадрат периода пропорционален кубу больших полуосей. И здесь важно не путать полуоси с радиусом, иначе у тебя получится хуйня!

Когда мы спиздили у планеты скорость - она не может резко телепортироваться на другую орбиту, а значит её апоцентр остаётся прежним. А вот перицентр уменьшается, и орбита превращается в эллиптическую. Орбитальный период при этом нихуя не растёт, а наоборот уменьшается. Вблизи перицентра она при этом разгоняется, ибо потенциальная энергия там переходит в кинетическую, как в маятнике. Но общая энергия становится меньше.

Если это происходит долго и равномерно - будет казаться, что у круглой орбиты просто плавно уменьшается радиус. Но это нихуя не так. Мы всегда уменьшаем только нижний кусочек эллипса, находясь в верхнем, и период для эллипса считается совсем иначе. Поэтому и получается парадокс, что спутники на низких орбитах летают с бешеной орбитальной скоростью, а на высоких еле ползают. У высоких суммарной энергии больше, просто запасена она почти вся в потенциальной, и чем больше ты крадёшь кинетической - тем больше потенциальная высвобождается.
Аноним 16/07/23 Вск 17:32:36 758740 157
>>758736
>Если это происходит долго и равномерно - будет казаться, что у круглой орбиты просто плавно уменьшается радиус. Но это нихуя не так. Мы всегда уменьшаем только нижний кусочек эллипса, находясь в верхнем, и период для эллипса считается совсем иначе. Поэтому и получается парадокс, что спутники на низких орбитах летают с бешеной орбитальной скоростью, а на высоких еле ползают. У высоких суммарной энергии больше, просто запасена она почти вся в потенциальной, и чем больше ты крадёшь кинетической - тем больше потенциальная высвобождается.
Значит ли это, часто обнаружив систему, в которой есть планеты с явно выраженной закруглённостью/вытянутостью орбиты, при том что другие планеты не таковы в своих траекториях, то мы тем самым сможем уверенно сказать, что тут имело место обширное воровство гравитации через гравиманёвры великого числа малых или небольшого числа больших тел?
Послны чо ты там уебб Аноним 16/07/23 Вск 17:41:17 758742 158
Нашел черные звезды?
Аноним 16/07/23 Вск 18:03:08 758743 159
>>758742
Нормально ты наебенился, я смотрю.
Аноним 16/07/23 Вск 18:04:58 758744 160
>>758740
Не значит.
Во-первых, планеты с кривыми орбитами и от естественных причин получаются.
Во-вторых, чтобы из круглой орбиты образовался заметный эллипс - надо либо однократно ускорить гравиманёвром нечто хотя бы в 1% массы от планеты, либо миллионы (как бы не миллиарды) лет совершать гравиманёвр строго в одной точке орбиты и в одинаковом направлении и ни в каких других. Никакая разумная жизнь подобной хуйнёй заниматься не будет.
Аноним 16/07/23 Вск 18:54:38 758745 161
>>758744
>Никакая разумная жизнь подобной хуйнёй заниматься не будет.
Если только это не какая-нибудь удобная транзитная система с планетой-трамплином для переброса по некой межзвёздной магистрали, где чётко высчитана одна планета для одной цели - гравиманёвра
Аноним 16/07/23 Вск 20:42:46 758748 162
>>758745
Для межзвёздной магистрали имеют смысл гравиманёвры разве что вблизи чёрных дыр. Как только ты освоил хотя бы ТЯРД с тысячами км/с дельты, всё это крохобороство с гравиманёврами становится нахуй не нужно. Я уже не говорю про околорелятивистские скорости.
Аноним 16/07/23 Вск 21:10:43 758749 163
>>758748
Сюррелятивизм какой-то
Аноним 17/07/23 Пнд 00:43:06 758752 164
Если я двигаюсь ровно относительно расширения вселенной и засовываю в пустоту руку, я смогу обратно её вытащить или часть руки исчезнет?
Аноним 17/07/23 Пнд 01:28:36 758756 165
>>758748
Заебись маневр, пролететь 10 световых лет со скоростью 0.5 скорости света за 40 лет?
Аноним 17/07/23 Пнд 01:47:46 758757 166
>>758752
Щас ты уже никуда не сунешь, опоздал ты на расширение вселенной в первые секунды, теперь все поразлеталось но десятки млрд лет, да ещё с ускорением, так что вокруг горизонт событий, который давно уже убегает со скоростью больше световой.
Аноним 17/07/23 Пнд 06:19:11 758761 167
image.png 255Кб, 500x275
500x275
>>758093
Не светильник, а дырка.
Аноним 17/07/23 Пнд 08:40:50 758772 168
>>758752
>Если я двигаюсь ровно относительно расширения вселенной
Это как? У расширения нет выделенных направлений. Ты и так каждую секунду "движешься относительно расширения", потому что Вселенная расширяется во всех своих точках каждую секунду
Аноним 17/07/23 Пнд 11:23:53 758778 169
Video by annama[...].mp4 2557Кб, 576x1024, 00:00:15
576x1024
54 года назад летал Apollo 11. Оправдывайтесь.
Аноним 17/07/23 Пнд 11:32:05 758780 170
>>758778
Раммшатайн снимали свой клип на Луне и не нашли следов Аполло.
Аноним 17/07/23 Пнд 11:56:03 758783 171
>>758757
>>758772
Из вопроса очевидно что у меня есть возможность засунуть руку за пределы расширения вселенной, главное не то как я это сделаю, а что будет если я засуну руку. Может она упрёться об твердь?
Аноним 17/07/23 Пнд 11:56:26 758784 172
>>758783
Упрётся* быстрофикс
Аноним 17/07/23 Пнд 11:58:16 758785 173
>>758783
У расширения Вселенной нет пределов.
Аноним 17/07/23 Пнд 12:12:34 758788 174
>>758785
Да это не ответ, можно представить гипотетически что есть
Аноним 17/07/23 Пнд 12:15:04 758789 175
>>758788
Дебил что ли? Если есть предел то упрешься, если нет тогда там где твоя рука там будет расширена вселенная за счёт атомов твоей руки. Не лезь она тебя сожрет
Аноним 17/07/23 Пнд 13:27:12 758799 176
>>758783
>возможность засунуть руку за пределы расширения вселенной
Не существует никаких "пределов". Твой вопрос изначально лишен смысла, потому что базируется на абсурдных посылках
Аноним 17/07/23 Пнд 14:18:42 758800 177
>>758789
Ну вот, наконец то нормальный ответ.
Аноним 17/07/23 Пнд 15:18:50 758804 178
d2b68d14bea9fb6[...].jpg 377Кб, 735x825
735x825
Если 2 космических туриста типа Эм и Жо займутся на орбите сегзом (прямо бенисом в багину), начиная от ерекции и заканчивая кууумингом в ваджайну, то... как это скажется на космосекстуристах?
Ведь в условиях невесомости сильно меняются токи всех соков в организме из-за пропадания привычно для человека при н.у. вектора гравитации. То есть, биохимия самого оргазма может измениться и не дать ощущения, или дать усиленно, или дать вообще не те. Да и особенности поведения жидкостей такого рода в пост-коитальной ситуации тоже вызывают определённое любопытство.
Аноним 17/07/23 Пнд 15:36:44 758807 179
>>758752
Тебе только позволь совать руки куда попало, ты и бога за четырехмерную пятку схватишь.
Аноним 17/07/23 Пнд 15:43:24 758808 180
>>758804
Такие эксперименты несколько раз проводились на МКС, но результаты засекречены ещё впереди.
Аноним 17/07/23 Пнд 19:19:40 758823 181
>>758804
Ничего не будет, самцов кормят кетоконазолом чтоб не стояло.
Аноним 17/07/23 Пнд 19:42:18 758824 182
1689612121794.mp4 3747Кб, 640x480, 00:00:45
640x480
Гравитация это англосаксонское изобретение.
Аноним 17/07/23 Пнд 22:41:02 758835 183
Анон, помоги понять такую штуку.
Говорят в 2024-2025 году на Землю упадёт астероид.
Смотрел я видео одного жидорептилоидного равина, ищущего глубокие смыслы в том, что и как написано в Торе, и что, мол, там на обозначенные годы указаны страшные события для Израиля и/или Японии.
Ну, предположим, так и будет и что Израиль и Япония находятся в поясе Земли, лежащем под траекторией падения астероида и его осколков с некоторым допуском по ширине.
А есть ли какие-нибудь онлайн браузерные крутилки где можно составить линии и посмотреть, находится ли эти две страны в одном поясе и какие ещё страны могут попасть под удар
Аноним 17/07/23 Пнд 23:07:41 758841 184
>>758835
> что и как написано в Торе, и что, мол, там на обозначенные годы указаны страшные события
Помню такую же хуню про Календарь Майя - не взлетело.
Аноним 17/07/23 Пнд 23:23:31 758844 185
>>758835
>Говорят в 2024-2025 году на Землю упадёт астероид.
Они каждый год падают, по нескольку штук.
>ищущего глубокие смыслы в том, что и как написано в Торе, и что, мол, там на обозначенные годы указаны страшные события для Израиля и/или Японии.
Они каждый год обещают адъ и израиль, а в итоге получается только израиль.
>А есть ли какие-нибудь онлайн браузерные крутилки где можно составить линии и посмотреть, находится ли эти две страны в одном поясе и какие ещё страны могут попасть под удар
Да.
Аноним 18/07/23 Втр 07:30:18 758858 186
image.png 539Кб, 990x590
990x590
image.png 183Кб, 720x405
720x405
>>758844
>Они каждый год обещают адъ и израиль, а в итоге получается только израиль.
Соглы, тут соглы. Но вот недавно у берегов Чили выловили глубоководную рыбину, которая сулит катаклизмы и катастрофы черех какое-то время, от пары дней до нескольких лет, после обнаружения, а на днях рыба-ремень - сельдяной король, "рыба апокалипсиса" - обнаружилась у берегов Тайваня. И там, типа, тоже считается она предвестником катастроф. Так что, кто знает? Может это предчувствие астеройда в 24-25. Или, как там в Апокалипсисе:
Второй Ангел вострубил, и как бы большая гора, пылающая огнем, низверглась в море; и третья часть моря сделалась кровью, и умерла третья часть одушевленных тварей, живущих в море, и третья часть судов погибла.
Может, это вот оно самое и есть. За всё хентайную порнуху, которой японцы отравили разумы честных анончиков, за футанари, лоли и копро манги и хентайный мультики накажут трусонюхов
>Да.
Пизда.
Дай ссылку, что ли
Или - если есть тут не рукожопые калякеры-малякеры, нарисуйте, пожалуйста, на плоской карте волну а-ля пикрил, чтобы было видно наглядно потенциальную зону поражения (само собой, через Японию и Жидорептилоидостан).
Аноним 18/07/23 Втр 08:37:56 758862 187
>>758858
Пиздуй в /re и там обсуждай всякие древние жидовские сказки. Здесь доска с другой тематикой
Аноним 18/07/23 Втр 11:14:41 758872 188
image.png 206Кб, 500x350
500x350
>>758858
Открываш гугол ёрф и смотришь. А куда смотреть? На планету. Она вся в зоне поражения, нет каких-то особоых астероидопадежных зон, они где угодно падают. Напомню, что последние два камня крупнее пяти метров ебнулись в России, а не в япониях-израилях.
Аноним 18/07/23 Втр 13:45:05 758889 189
Как изменилась бы наша жизнь, если бы скорость света в вакууме была на 10 или 20% больше или меньше?
Аноним 18/07/23 Втр 14:05:05 758892 190
>>758862
>Здесь доска с другой тематикой
Так я сюда не за религиозными дискуссиями пришёл
>>758872
Ну, открыл
https://earth.google.com/web
Где здесь можно провести плоскость сечения через центр земли заданными мною точками, дабы посмотреть, через какие места, страны и города это плоскость проходит?
Аноним 18/07/23 Втр 14:06:45 758893 191
>>758889
>Как изменилась бы наша жизнь, если бы скорость света в вакууме была на 10 или 20% больше или меньше?
Читай Лю Цисиня, трилогию "Задачи трёх тёл", в частности последнюю книгу трилогии "Вечная жизнь смерти" - там автор рассказывает, что будет, если играться такими константами, как скорость света
Аноним 18/07/23 Втр 15:43:31 758896 192
>>758892
Не понимаю зачем тебе эта хуйня. Вся земля практически равномерно под шансом падения.
Аноним 18/07/23 Втр 17:09:11 758898 193
>>758896
>Не понимаю зачем тебе эта хуйня. Вся земля практически равномерно под шансом падения.
Да нихуя. Если бы вот мы знали точно, что есть никой объектоид, который движется по плоскости экватора Земли, то мы мыогли бы точно сказать, в каких зонах он может упасть - по экватору, логично же. Ну и зона поражения в сотенку другую км к северу и югу от настоящего экватора. Но это же слишком просто, ведь Земля прецессирует, колобродит, вращается на боку с небольшим наклоном. Значит, траектория такого тела попадает на плоскость, проекция которой на Землю при переносе на плоскую карту превращается в такие вот волны. Это определение двух заданных для создания плоскости сечения точек (в смысле проверки того, как бы подобный график выглядел на плоской карте) позволит понять, задет ли страну, регион и город потенциально определённое событие, а если нет, то смотяр на вертояные места падения, думать о том, как это может сказаться, и к чему это может привезти в общемировом плане в перспективе климата, политиик, катастроф и т.д. ближайшего после падения времени. Ведь одно дело, если ёбнется в океан, другое дело - куда-нибудь на Японию, восседающую на тектоническом разломе, третье дело - в песках мусульман, которые могут признать это новой Меккой или каким-нибудь знаком свыше, или вообще ёбнет по какой-нибудь гига-кальдере с дохуищем сернистого газа или по АЭС

Нужно определить возможные места падения, что определить уровень тряски и грядущего визга
Аноним 18/07/23 Втр 17:27:42 758900 194
>>758898
>возможные места падения
Сейчас скажу: без конкретных цифр и орбитальных параметров эти места - ГДЕ УГОДНО. с конкретными же цифрами уже посчитали бы где что упадет, и тебе не надо было бы этим заниматься.
Аноним 18/07/23 Втр 17:55:24 758904 195
>>758900
Не лишай мой разум пищи для размышлений, сука!
Аноним 18/07/23 Втр 19:24:49 758908 196
>>758904
размышляй делая конкретные вычисления
Аноним 18/07/23 Втр 19:27:45 758909 197
Ну че, сильно мир изменился после открытия бозона Хиггса?
Аноним 18/07/23 Втр 19:30:52 758910 198
Аноним 18/07/23 Втр 19:35:44 758912 199
>>758909
Мир с тех пор ебанулся на отличненько. Градус ебанутости повышается с ускорением.
Аноним 18/07/23 Втр 20:57:19 758920 200
что больше весит, килограмм очарованного кварка или килограмм подзалупного творожка фейнмана?
Аноним 18/07/23 Втр 21:14:27 758923 201
image.png 805Кб, 1280x720
1280x720
Насколько реален Минмус из огурцов?
То есть, на безатмосферной лёгковесной планете в зоне жидкой воды чтобы был лёд, пускай даже не совсем из воды, но явно выделяемый от прочих побережных, горных и степных элементов местного рельефа
Аноним 18/07/23 Втр 21:20:31 758924 202
Если построить гигантский корабль в виде сосиски и массой земли, он схлопнется в шар под собственной гравитацией или шар формируется, когда тело разогрето и пластично? Есть какая то граница массы, когда тело из-за гравитации начнет плавится в центре и приобретать шарообразную форму?
Аноним 18/07/23 Втр 21:22:24 758926 203
>>758924
когда твердое тело из-за гравитации начнет
Аноним 18/07/23 Втр 21:23:38 758927 204
image.png 38Кб, 200x200
200x200
>>758920
Килограмм творожка, ведь на тебя еще будет давить стыд за то что ты его выскребал.

>>758923
Вовсе не реален, летучая жижа испарится в космос. Может там вместо воды (льда) чугунина какая, тогда может быть.

>>758924
Схлопнется в шар.
Есть граница, и определяется сопроматом, но на массах земли вроде материалов нет чтоб удерживали формы, все эти металлургии жижею стекают под собственной тяжестью.
Аноним 18/07/23 Втр 21:32:35 758928 205
>>758924
> Есть какая то граница массы, когда тело из-за гравитации начнет плавится в центре и приобретать шарообразную форму?
Мне нравится этот вопрос, но хочу его переиначить.
А что будет если мы возьмём гигантский кусман воды, гигакаплю, просто химически чистую H2O тучу воды и поместим на орбите какой-то планеты. Какого размера должна быть такая планета, чтобы под давлением внешней воды, такая планета сформировала бы ядро суперплотной воды?
какие были бы свойства у такого ядра да и вообще у планеты? Магнитное поле было бы хоть?
Аноним 18/07/23 Втр 21:43:38 758930 206
Аноним 18/07/23 Втр 21:45:07 758931 207
>>758924
>начнет плавится
Пвнинское образование, итоги
Аноним 18/07/23 Втр 22:10:53 758932 208
>>758931
>Доебаться до формы
>Не ответить по содержанию
>Пиздеть про образование
Аноним 18/07/23 Втр 23:24:38 758937 209
>>758930
Это значит, что у запутанных частиц нету и не может быть никаких скрытых параметров, содержащих данные о взаимной суперпозиции, которые бы объясняли все эти квантовые чудеса в рамках какого-то нормального физического механизма, работающего в независимой от наблюдателя единой для всех реальности.

А это в свою очередь значит, что нет никакой единой для всех реальности. И что Эверетт и другие шизы были правы, и все эти ветвления параллельных миров действительно происходят.
Аноним 19/07/23 Срд 08:40:22 758949 210
>>758937
>А это в свою очередь значит, что нет никакой единой для всех реальности. И что Эверетт и другие шизы были правы, и все эти ветвления параллельных миров действительно происходят
Хуйню не неси и не вводи людей в заблуждение. Это означает лишь отсутствие локального реализма, т.е. нелокальность - отсутствие локальных скрытых переменных. Поебень про ветвление миров и "отсутствие единой для всех реальности" - это уже шизофантазии
Аноним 19/07/23 Срд 09:00:20 758950 211
>>758823
А тянам пизду заваривают, но они успевают натрахаться перед полётом. А сиськи у них забирают и в сейф кладут на время полёта, а когда они возращуются на землю и забирают их - те без кровоснабжения плоские как блины становятся, поэтому космонавток с большими сиськами не бывает.
Аноним 19/07/23 Срд 09:21:12 758953 212
>>758949
Ну давай, объясни этот механизм без локальных скрытых переменных и без ветвлений параллельных миров. Между двумя частицами с нихуя образовалась стабильная червоточина наверное, и они через неё спинами ебутся. Или всё давно посчитано на пеке у боженьки, поэтому кошмарное дальнодействие в этой симуляции есть, но смотреть на него нельзя, иначе бох накажет.
Аноним 19/07/23 Срд 10:44:50 758957 213
>>758953
Иди нахуй чмонька необразованная.
Аноним 19/07/23 Срд 11:27:08 758961 214
>>758953
>объясни этот механизм без локальных скрытых переменных
Я вообще-то и говорю о том, что данное явление имеет нелокальную природу, что и подтверждается экспериментами. В глаза долбишься?
>без ветвлений параллельных миров
А эти ветвления параллельных миров сейчас с нами в комнате?
>Между двумя частицами с нихуя образовалась стабильная червоточина наверное
Нет. Просто квантовая запутанность - нелокальное явление: пара запутанных частиц представляет собой единый квантово-механический объект с одной волновой функцией. Декогеренция одной из частиц приводит к изменению всей волновой функции, а значит и к автоматической декогеренции второй частицы. При этом между частицами не происходит никаких взаимодействий, они не обмениваются между собой никакими агентами, переносящими информацию, - в этом заключается суть явления нелокальности. А отсутствие скрытых переменных означает, что до измерения пары запутанных частиц не существует в принципе никакой возможности как-то узнать их состояние, "подглядеть" его - состояние как таковое рождается лишь в момент непосредственного измерения, т.е. в результате декогеренции; оно не предопределяется ничем заранее. Вот и все. И никаких тебе червоточин, никакой непроверяемой антинаучной шизохуйни с ветвящимися сущностями в виде гномика
>Или всё давно посчитано на пеке у боженьки, поэтому кошмарное дальнодействие в этой симуляции есть, но смотреть на него нельзя, иначе бох накажет
Это тебе в /re надо с такой хуйней. Что-то нынче частенько на доску стали захаживать долбоебы с воспаленным мозгом, которым так и чешется попиздеть про бога на доске с совершенно другой тематикой. Летнее обострение, наверное
Аноним 19/07/23 Срд 12:45:30 758962 215
>>758961
Вся эта хуяльность-нелокальность слишком походит на то, как древние папуасы объясняли суть молнии, наводнения, землетрясения, особо злючючего пердежа - это бог молнии ебошит, это бог подарил возможность злобно бздеть, ангелы напились божественного пиваса и поэтому дождь идёт. Просто есть вещи которые не смогут объяснить даже мочёные. Я не сторонник бога-хуёга, я целиком и за научное объяснение, но, согласись, некоторые вещи выглядят нелепо, хоть типа и "объясняют всё", тн виртуальные чястицы, измерения выше третьего, большой взрыв... Ну ведь просто же бабло пилят суки, умело пилят, причём. Очень, блять, умело.
Аноним 19/07/23 Срд 15:06:51 758973 216
>>758962
Ну и ты пили вместе с ними, хули бухтеть?
А хочешь истину - так выбрось из головы весь пиломатериал и начни с нуля - подвергая всё сомнению.
Аноним 19/07/23 Срд 16:17:50 758989 217
>>758962
>некоторые вещи выглядят нелепо, хоть типа и "объясняют всё", тн виртуальные чястицы, измерения выше третьего, большой взрыв... Ну ведь просто же бабло пилят суки, умело пилят, причём. Очень, блять, умело.
А ещё микробы, радиоволны, круглая земля. Чего толко не навыдумывают,что бы простого рабочего человека обокрасть!
Аноним 19/07/23 Срд 18:19:12 758996 218
>>758962
Это в /zog, ты не на той доске торчишь, шиз
Аноним 19/07/23 Срд 18:37:11 758997 219
>>758973
>>758989
Так я и говорю, пилят оче умело, я завидую. Истину хуй постичь, на ней бобло рубят со времён сраного христа. Христа рубят, бобло пилят((((
Аноним 19/07/23 Срд 18:39:26 758998 220
>>758996
Не кавитируй, прочитай внимательно, я за науку, но против ебанутых научных идей.
Аноним 19/07/23 Срд 19:41:49 759005 221
>>758893
> последнюю книгу трилогии "Вечная жизнь смерти"
Я первые 2 прочитал, а эту не стал, так как по описанию показалась лишней. Эпопея с трисолярием же кончилась уже, поэтому выглядит как попытка срубить бабла на имени.
Теперь наверно почитаю.
Другой анон
Аноним 19/07/23 Срд 22:41:50 759020 222
А можно ли добывать тяжелые элементы или хотя бы железо хуяря по поверхности нейтронной звезды какими-нибудь стержнями Тора с релятивистской скоростью и собирать осколки, или с какой скоростью ни метай - ничего не отлетит?
Аноним 19/07/23 Срд 22:42:43 759021 223
>>759020
И тут же вдогонку - если так поколупать НЗ, можно ли ее раскукожить в итоге получается?
инб4 не нужно, страпельки вылезут
Аноним 20/07/23 Чтв 00:03:41 759024 224
>>759020
можно

только вёдер побольше захватить надо, мне в прошлый раз не хватило и больше половины в космос улетело
Аноним 20/07/23 Чтв 01:06:53 759026 225
>>759020
Нейтронная звезда релятивистский объект. У нее ньютоновская гравитационная энергия сопоставима с энергией покоя, поэтому ее нельзя рассматривать как обычный объект. Чтоб оторвать от нее какой-то значимый кусок, нужно изменить ее гравитацию. Короче нужно кидать в нее компактные массивны объекты - такие же нейронные звезды или черные дыры.

Чтоб обнажить ядро нейтронной звезды, где всякие страшные страпельки и кварковая малафья, нужно ее разрушение приливом от черной дыры малой массы, существенно ниже чем солнечная. Такие пока что не встречаются. Боженька удачно подкрутил параметры во Вселенной.

Другой момент, в силу релятивистских эффектов, обычное вещество не может существовать в ее окрестностях. Атомы тупо разрывает отдельно на электроны и ядра, которые в "вмораживается" в мощные магнитные поля. Энергия падения передается не веществу, а полям в окрестностям звезды. Максимум что можно "выбить" это вещество "вмороженное" в поле, а там преимущественно электроны и легкие ядра.
Аноним 20/07/23 Чтв 04:47:50 759030 226
>>759020
Это только в фильме Тор смелый и сексуальный блондинчик, а отправь его добыть килограмм нейтронного говна, тоесь от нейтронной звезды отколупнуть чють-чють - он мигом отмазы начнёт придумывать, что она не релятивистская, невозможно от неё отколупывать, там формулы за 5 класс не работают. Ага, верю, как же бля, ссыкло белобрысое. Как перед земляшками выябуваться - прям мастер. Я возьму акаес и приеду на мадагаскар, построю всех местных людоедов и даже лемуров. Вот то же самое.
Аноним 20/07/23 Чтв 09:12:01 759041 227
>>758998
Ты дурачок галимый. Говоришь вещи космического масштаба космической же глупости, не имея ни малейшего представления о предмете, о котором пытаешься рассуждать. На хуй
Аноним 20/07/23 Чтв 10:57:15 759045 228
1.mp4 1960Кб, 640x360, 00:00:25
640x360
Решил спросить в этом разделе. Вот этот дым от шин это канцероген? Или он безопасен и можно полной грудью?
Аноним 20/07/23 Чтв 11:24:29 759046 229
>>759045
Да все в этом мире сплошной канцероген. Можешь сразу ползти на кладбище
Аноним 20/07/23 Чтв 12:59:58 759064 230
Аноним 20/07/23 Чтв 15:03:05 759066 231
>>759064
>пук хрюк уиииаиииаиаии
Аноним 20/07/23 Чтв 17:33:45 759080 232
>>758961
>Просто квантовая запутанность - нелокальное явление: пара запутанных частиц представляет собой единый квантово-механический объект с одной волновой функцией. Декогеренция одной из частиц приводит к изменению всей волновой функции, а значит и к автоматической декогеренции второй частицы. При этом между частицами не происходит никаких взаимодействий, они не обмениваются между собой никакими агентами, переносящими информацию, - в этом заключается суть явления нелокальности.
Это означает, что я могу, находясь в 1 световом годе, дистанционно коллапснуть волновую функцию этого квантовомеханического объекта ещё до того, как частица попала в экран. И передавать таким образом информацию в реальном времени, обосрав всё ограничение на скорость света.
А значит, информация где-то передаётся. Внутри квантовомеханического объекта передачи информации нет, тогда где? Вот все эти приколы с параллельными мирами - есть перекладывание передачи информации на всю внешнюю реальность.
Аноним 20/07/23 Чтв 19:22:18 759094 233
>>758347
Да, но скорее всего искусственно
Аноним 20/07/23 Чтв 19:30:20 759096 234
>>759080
> И передавать таким образом информацию
нет способа передать так информацию
ты можешь получить информацию о функции частицы в световом годе (при условии что она до сих пор ни разу ни с чем за время разлёта не пересекалась), но не передать
Аноним 20/07/23 Чтв 19:51:41 759097 235
Аноним 21/07/23 Птн 01:22:16 759129 236
>>759026
То есть, я так пони, как ни бахай, уменьшить НЗ нельзя, и любая энергия которую затратишь на попытки колупать ее лишь увеличит ее массу?
Аноним 21/07/23 Птн 08:49:22 759136 237
>>759080
>ещё до того, как частица попала в экран
Какая частица в какой экран?
>И передавать таким образом информацию в реальном времени
Нет, никакой информации при квантовой запутанности не передается
>обосрав всё ограничение на скорость света
У тебя нет никаких агентов - переносчиков взаимодействий, которые бы бегали в пространстве между частицами. Явление нелокальное, никаких взаимодействий между частицами не происходит. Именно поэтому передача информации невозможна
>А значит, информация где-то передаётся
Нихуя не значит. Из чего ты это вообще вывел? Примерно почувствовал?
>тогда где
Нигде
>Вот все эти приколы с параллельными мирами
Все эти приколы - антинаучная хуйня и говно без задач. В науке просто предположения, которые ничем, кроме пустых слов, не подкреплены, не имеют никакого значения. Ты можешь предполагать хоть параллельные миры, хоть параллельные хуи, хоть розовых единорогов с зубастой пиздой на спине - все это не будет иметь никакого отношения к науке, а следовательно, будет являться беспонтовым пердежом в лужу, до тех пор, пока ты на основе какого-либо из этих предположений не построишь математическую модель с конкретными параметрами, дающую конкретные предсказания, которые можно проверить экспериментально и либо подтвердить ее, включив в научную картину мира, либо опровергнуть и выкинуть на свалку истории как неверное представление об устройстве мира. Именно поэтому многомировая интерпретация идет на хуй как ненаучное говно. Она не дает ни одного проверяемого предсказания, просто постулируя волшебную поебень с ветвлением реальности и ни на что при этом не опираясь, а потому для познания мира она совершенно бесполезна
>>759097
Что терпеть-то? Твою умственную отсталость?
Аноним 21/07/23 Птн 10:32:27 759140 238
001.png 14Кб, 1756x254
1756x254
>>759136
>Нет, никакой информации при квантовой запутанности не передается
Между Алисой и Бобом один световой год.
Посередине стоит источник, из которого в обе стороны непрерывно испускаются запутанные частицы.
И у Алисы, и у Боба есть экран и перегородка с двумя щелями, как у Томаса Юнга, на обоих экранах картинка со множеством полос.

Алиса хочет передать сигнал Бобу. Она ставит детектор возле одной из щелей. Волновая функция при этом коллапсирует, и на её экране возникают две полоски. Но поскольку волновая функция у частиц одна, значит и коллапсирует она как одна, значит две полоски появляются на обоих экранах. Боб это видит и понимает, что Алиса передаёт ему сигнал. Причём система симметрична, и Боб ей может ответить, и они могут общаться морзянкой в реальном времени.
Аноним 21/07/23 Птн 12:04:23 759146 239
>>759129
Вообще говоря можно.
Магнитные полюса(которых у нейтронный звезды может быть несколько) являются дорожкой по которой вещество может двигается в обоих направлениях игнорируя гравитацию. Если полюс достаточно горячий, то будет мощное истечение вещества(относительно массы звезды все равно маленькое и опять так выбрасываются легкие элементы). Почти как релятивистский джет при аккреции, но он ненаправленный, не равномерный и неправильной формы. Обычно такие выбросы вещества наблюдаются и молодых нейтронных звезд.
Короче посвети мощной йоба гига гамма лазерной указкой на магнитный полюс звезды и будет тебе счастье.
Аноним 21/07/23 Птн 12:20:59 759151 240
image.png 65Кб, 259x194
259x194
>>759146
Спасибо, попробую!
Аноним 21/07/23 Птн 15:38:07 759181 241
>>759140
Каким образом ты додумался до того, что с помощью такой хуйни тебе удастся использовать квантовую запутанность для передачи информации? Ты, кажется, не понимаешь, что такое квантовая запутанность, поэтому и допускаешь в своих рассуждениях детские ошибки.
Твой "мысленный эксперимент" не имеет отношения к реальности уже хотя бы потому, что не учитывает фундаментальных свойств квантовой запутанности - случайность и нелокальность. "Морзянкой" у них с помощью двухщелевого эксперимента с запутанными парами не получится общаться, потому что никто из них не может управлять состоянием частиц: состояние, как уже говорилось выше, рождается лишь в момент декогеренции и не зависит ни от чего. Тут играет свою роль случайность. Это во-первых. Ну и во-вторых, как уже опять-таки говорилось, между частицами не происходит никакого обмена переносчиками взаимодействий, а передавать информацию в нашей Вселенной можно только посредством переносчиков взаимодействий. Здесь в игру вступает нелокальность. Эти фундаментальные факторы квантовой запутанности полностью исключают какую-либо возможность использовать данное явление для передачи информации, включая твой глупый мысленный эксперимент
Аноним 21/07/23 Птн 17:32:00 759200 242
Сап, пачаны. Такой вопрос. А как мы оказались там, где мы есть. Я про материю. Мы знаем, что вселенная появилась примерно 13,8 миллиардов лет назаж в большом взрыве
В одной точке.
Ничто не может двигаться быстрее света
Джеймс Уэбб видит объекты на расстоянии 13,74 милларда световых лет.
Как же материе распространилась так далеко
Ну и мы же видим сверхдалекие объекты со всех сторон, как я понимаю. Это как материя так распространилась???
Аноним 21/07/23 Птн 17:40:30 759202 243
>>759200
Быстрее скорости света пространство расширилось.
Аноним 21/07/23 Птн 18:05:53 759208 244
>>759200
>В одной точке
Не было никакой "одной точки". Большой взрыв - не взрыв, а начало расширения Вселенной. Наша Вселенная - это трехмерное пространство. Когда оно возникло, оно имело микроскопический объем, равный планковской длине - 10^-33 см. Затем оно начало экспоненциально расширяться, началась инфляционная эпоха - это и есть то, что в современной космологии называют Большим взрывом. Расширение происходило во всех точках пространства, во всем его объеме, т.е. оно имело изотропный характер. По окончании эпохи инфляции в результате квантовых флуктуаций вакуума родились элементарные частицы - Вселенная заполнилась веществом. Так же, как и расширение пространства, рождение частиц происходило по всему объему Вселенной, во всех ее точках - никакого "распространения материи" никуда не происходило, вещество сразу возникло во всех частях пространства одновременно, расширяясь вместе с пространством
Аноним 21/07/23 Птн 20:30:49 759217 245
>>759140
Лол . Я понял в чем ты лузер.
Когда Алис попытается поставить две щели перед запутанной частицой, то она никак не повляет на частицу. Влияние произойдет только когда ты попытаешься узнать ее спин (вверх или вниз) при измерении вдоль одной оси конечно.
Допустим Алис измерила вверх. Боб измерил и получил всегда противоположный - вниз. Алис никак не может повлять на исход и Боб лишь только знает, что у алис спин вверх. Результат измерение спина - чистый рендом, его не предсказать.
Короче на этом коммуникацию не построишь.
Если бы Алис крутила спин сама по желанию как хочет и Боб бы в реальном времени видел это кручение - то да. Но проблемка в том, что мы не можем крутить спин частицы как колесо радиоприемника, и что после одного взаимодействия (измерения спина) эта сцепленность еще и нарушается (один раз замерил - ушла сцепленность).

Так что к сожалению хуй нам, а не сверхсветовая связь. С другой стороны сверхсветовая связь была бы еще хуже. Это значило бы, что мы можем влиять на события в прошлом, а это приводит к неразрешимым парадоксам. Или пришлось бы как то обьяснять, что скорость света не всегда одинакова для всех систем отсчета, что уже доказакно как бы что одинакова. Короче как не крути, но сверхсветовая связь это плохо.

По поводу передается инфа или нет, и твой спор с шарящими анонами - это больше игра в определения. Тебе аноны сказали же, что ПО ОПРЕДЕЛЕНИЮ инфа - это какая то измеряемая характеристика, которая передается благодаря одному из 4 взаимодействий. Просто мы так решили. Я так сказал.

Запутанность - это явление на грани научного прогресса. Мы не знаем как это работает. Возможно для "квантовых систем" действительно пространство не является преградой.
Или может быть в более широком смысле пространство - это лишь выдумка людей и пространства не существует! Существует лишь отношения между обьектами. Пространство же мыслительный конструкт, который помогает нам интуитивно понимать многие вещи.
Или может быть, что действительно есть некий особый канал "квантовой спецсвязи", который мы пока не поняли как его понять.
Вариантов дохуя, но все это лишь спекуляции.

Мне нравится больше всего идея супердетерминизма.
Он утверждает, что все в Вселенной было предопределено с самого начала, и что все, что происходит, является неминуемым результатом начальных условий. Таким образом, кажущиеся "действия на расстоянии" на самом деле являются просто продолжением этого начального состояния, и, следовательно, никакой "информации" не передается между запутанными частицами.
Аноним 21/07/23 Птн 20:34:18 759219 246
>>759136
Терпи свою анальную нелокальность.
Аноним 21/07/23 Птн 21:03:02 759225 247
>>759208
>Большой взрыв - не взрыв, а начало расширения Вселенной. Наша Вселенная - это трехмерное пространство. Когда оно возникло, оно имело микроскопический объем, равный планковской длине - 10^-33 см. Затем оно начало экспоненциально расширяться, началась инфляционная эпоха

В дурку этого!
Аноним 21/07/23 Птн 23:11:27 759233 248
>>759200
ты когда нибудь пробовал пернуть в воде?

бдзюдьку похуй на то что вода плотная, он как гигачад говорит мне похуй да) и просто раздвигает плотную водичку своими чадогазиками (но с тем отличием, что мы в курсе что такое вода, а вот что представляет собой запределье вселенной — нет)

учитель физики средней школы
Аноним 22/07/23 Суб 03:03:27 759271 249
>>759217
>Когда Алис попытается поставить две щели перед запутанной частицой, то она никак не повляет на частицу. Влияние произойдет только когда ты попытаешься узнать ее спин (вверх или вниз) при измерении вдоль одной оси конечно.
Шизик, Алиса не меряла спин. Алиса меряла координату. Она может получить либо "щель 1", либо "щель 2". Если измерение не делать - частица проходит через обе щели.

Частица прошла через одну щель (похуй через какую) - это логическая единица.
Частица прошла через обе щели - это логический ноль.

>Боб измерил и получил всегда противоположный - вниз.
Боб нихуя не измерял. Боб просто смотрит на экран, где просходит то же самое, что и на алисином экране, хотя Боб возле своих щелей детекторы не ставил.
Я повторюсь, Бобу глубоко похую, через какую именно из двух щелей прошла частица после того, как Алиса измерила её пару. Бобу важно, прошла ли она через одну или через обе.

При этом ситуация, при которой у Алисы частица прошла через одну щель, а у Боба через обе, невозможна. Даже с учётом исчезновения сцепленности в момент измерения. Потому что в этом случае получилось бы, что частица Боба сохранила незатронутый измерением импульс, и этот импульс Боб мог бы измерить, а потом передать результаты Алисе по обычному каналу. И тогда получилось бы, что они наебали принцип неопределённости Гейзенберга, узнав со 100% достоверностью и координату, и импульс. Это так называемый ЭПР-парадокс. Опыты по ЭПР-парадоксу показали, что принцип неопределённости Гейзенберга не наёбывается, а конкретный параметр в момент измерения возникает у обоих частиц сразу. Возник этот параметр или не возник - на обоих экранах видно, сразу, мгновенно, независимо от расстояния. И на этом можно построить сверхсветовую связь.
Аноним 22/07/23 Суб 08:57:14 759285 250
Что там с J1839–10? Я пробежался по статьям и не увидел гипотезы что это двойная звезда. Раз слишком медленно для магнетара/пульсара, не может ли быть что это двойная звезда и эта парность обеспечивает такую пульсацию? Может НЗ или БК собирают говно с ближайшей звезды с постоянным потоком и бахают, или ЧД рядом и из-за колебаний орбиты только с такой периодичностью на нас светит?
Аноним 22/07/23 Суб 09:29:22 759286 251
>>759219
За щеку тебе натерпел, проверяй
>>759225
>пук
Аноним 22/07/23 Суб 11:38:44 759288 252
Я не понимаю сидерический месяц.
"Период, за который Луна вращается вокруг Земного шара относительно неподвижных звезд, называется сидерическим месяцем."
Но звёзды же не неподвижны, некоторые из них видны только зимой, только летом, в одном полушарии. Как же тогда определяют месяц?
ХУЙ 22/07/23 Суб 19:16:18 759314 253
Парни, черная материя и черная энергия, поясните на пальцах, это типа как НЛО?
Аноним 22/07/23 Суб 19:28:47 759315 254
>>759200
>Мы знаем, что вселенная появилась примерно 13,8 миллиардов лет назаж

Моченные сам нихуя не знают, возномжно вселенной триллионы лет
Аноним 22/07/23 Суб 19:42:47 759319 255
пук.mp4 633Кб, 404x720, 00:00:08
404x720
Аноним 22/07/23 Суб 20:00:09 759320 256
>>759314
Это названия для того что отвечает за наблюдаемые галактические орбиты (видимой массы недостаточно с учетом классической гравитации) и того что отвечает за расширение вселенной (тут хуй знает что вообще движет вселенную).
Грубо говоря - это "хуй знает"ы. Так что да, это типа как НЛО, они тоже так называются потому что неопознаны.
Аноним 22/07/23 Суб 20:07:48 759321 257
Аноним 22/07/23 Суб 23:27:15 759331 258
больно ли умирать, падая на нейтронную звезду? есть способы предотвращения?
Аноним 22/07/23 Суб 23:34:05 759332 259
>>759200
Не знаем, скорее всего Вселенная стационарная и живёт намного дольше
Может, просто у нас тут постулат и фанатики, борющиеся со лженаукой
Да
Они там и зародились, во Вселенной идёт вялотекущий процесс образования материи и её аннигиляции
Нет
Аноним 23/07/23 Вск 00:02:15 759334 260
>>759331
Если не помрешь от облучения на подлете, то помрешь довольно быстро от других причин.
В первую очередь у нейтронные звезд чудовищные магнитные поля, настолько сильные, то атомы не могут существовать. И это еще не долетая по поверхности звезды. Бонусом идет атмосфера нейронной звезды, которая находится в магнитной ловушке. Плотность у нее может достигать значительно больше чем у воды. Столкновение с таким тебя мгновенно испарит.
Во вторых приливные силы, они не такие пиздецовые как у черные дыр, но тебе от этого легче не станет. Даже если ты попадешь на остывшую нейтронную звезду и мимо магнитного полюса, то тебе оторвет ноги, а бока вдавит вовнутрь. Прилив настолько сильный, что способен рвать химические связи. Тебя разорвет на атомы и их тоже.
Аноним 23/07/23 Вск 00:29:52 759335 261
>>759271
Если я не ошибаюсь, квантово запутываются только спины двух частиц и разрушается квантовая запутанность только при измерении спина одной из частиц. То есть детектор, замеряющий через какую щель пролетела первая частица, не должен влиять на вторую частицу, так как при этом не измеряется спин. Или я что-то не так понял?

мимо другой анон
Аноним 23/07/23 Вск 04:58:38 759337 262
>>759334
А что быстрее распидорасит, прилив или магнитные поля?
Аноним 23/07/23 Вск 07:09:05 759338 263
>>759208
Как могло столько материи сжаться в одну точку? Это невозможно. И где эта точка висела? Только не говори, что нигде. Вокруг неё что-то было.
Аноним 23/07/23 Вск 07:13:25 759339 264
>>759271
Получается, так можно мгновенно будет передавать информацию через огромные расстояния?
Аноним 23/07/23 Вск 08:11:59 759340 265
>>759338
Найс троллинг тупостью
>>759314
>черная материя и черная энергия
Таких не существует. Существуют темное вещество и темная энергия. Темное вещество - просто вещество, которое не испускает излучение и не взаимодействует с ним. По своим свойствам оно похоже на известные нам нейтрино.
Темная энергия - это вакуум с ненулевой плотностью энергии. Его свойствами обусловлено ускоренное расширение Вселенной
Аноним 23/07/23 Вск 08:24:50 759341 266
>>759340
Почему троллинг?
Аноним 23/07/23 Вск 10:48:00 759344 267
>>759337
Магнитные поля.
Она будут сильны ОЧЕНЬ долгое время(на порядки больше нынешнего возраста Вселенной).
Аноним 23/07/23 Вск 11:41:41 759345 268
>>759340
>>759320
Спасибо.

А вот гипотетически, если бы мы встретились с темным веществом, как бы оно выглядело? Типа черная черная нефть вязка?
Или это понятие размазано на парсеки?

Ну и с энергией также, типа сферы в фильмы Сфера по Крайтону?
Аноним 23/07/23 Вск 12:14:05 759350 269
>>759335
>Если я не ошибаюсь, квантово запутываются только спины двух частиц
Квантово запутывается абсолютно всё, что связано законами сохранения. Если в результате квантового события родилась пара частиц - они будут запутаны по импульсу, причём как по скаляру, так и по вектору. Каким бы ни был импульс первой частицы - вторая обязана лететь в строго противоположном направлении с такой же скоростью, иначе бы нарушался закон сохраения импульса. Со спином просто несколько удобнее работать, но это не единственный параметр.

>>759339
Получается что можно. Но учёные особо пиздеть об этом не любят потому что там целая цепочка парадоксов возникает, способных поломать научную картину мира нахрен.
Аноним 23/07/23 Вск 14:10:08 759356 270
>>759345
Оно бы никак не выглядело, в этом его суть, что оно с э/м не взаимодействует никак, только гравитационно, и, возможно, слабым взаимодействием.
Аноним 23/07/23 Вск 14:44:25 759360 271
>>759356
Пиздец какой-то.

То-есть, слабую гравитацию бы почувствовали при помощи инструментов, и все.

Это конечно пиздец!

Спасибо анон!
Аноним 23/07/23 Вск 14:52:54 759361 272
>>759208
А как тогда неоднородности во вселенной объяснить, если она из такого микроскопического объема расширялась? И что было ДО стадии расширения?
Аноним 23/07/23 Вск 15:13:29 759363 273
>>759361
Значит, это не такой уж микроскопический объём, и внутри его были ещё какие-то структуры.
Аноним 23/07/23 Вск 15:38:43 759364 274
>>759314>>759360
Тёмная материя и тёмная энергия это математический костыль, чтобы натянуть кривую вращения галактик и красное смещение на представления о космологии середины 20 века.
Вся эта космология уже по пизде пошла посредством обнаружения галактик с высокометалличными звёздами на самом краю пузыря Хаббла, практически сразу же (как считается) после Большого Взрыва, чего быть никак не могло. https://3dnews.ru/1090132/uchyonie-vozrodili-gipotezu-stareniya-sveta-vselennaya-moget-okazatsya-vdvoe-starshe-chem-schitaetsya
Уже и гипотезу старения света заново откопали (один шаг от того, чтобы эфир откопать), а значит все оценки приведённого расстояния до галактик могут пойти по пизде, а за ними и модели гравитации могут пойти по пизде.

Короче, надо поменьше выёбываться с космологическими экстраполяциями и побольше работать над реальными инструментами, может тогда и костыли не понадобятся.
Аноним 23/07/23 Вск 16:13:14 759367 275
>>759364
Это не костыли, уёба, это названия для наблюдаемых феноменов, у которых несколько возможных объяснений, какие-то "костыли", какие-то - переработка существующих теорий.
ТМ ничего не объясняет и не должна, заебали со своей тупостью.
Аноним 23/07/23 Вск 16:37:09 759369 276
>>759367
Это именно что костыли. Если ты, захотев подрочить с утра, обнаружишь между ног пустое место, то отсутствие хуя будет феноменом, а ночной гномик-хуекрад - выдуманной сущностью в попытке объяснить этот феномен. И весь остаток жизни ты посвятишь поискам ночного гномика-хуекрада, изучая места его возможного обитания и строя гипотезы относительно гномьей анатомии. Хотя может быть и не было никакого гномика-хуекрада, а ты на самом деле всё это время болел проказой, и хуй сам отвалился.
Аноним 23/07/23 Вск 17:22:13 759372 277
>>759345
>как бы оно выглядело
Никак, оно не испускает излучение и не взаимодействует с ним. Поэтому увидеть ты его никак не сможешь, оно полностью прозрачное
>Ну и с энергией также, типа сферы в фильмы Сфера по Крайтону?
Фильм не смотрел, но темная энергия - это не вещество. Это вакуум с ненулевой плотностью энергии. Что такое вакуум? Пустое пространство. Но в квантовой теории вакуум не является пустотой, в нем постоянно происходят нулевые колебания всех полей. В итоге вакуум имеет плотность энергии, отличную от нуля. Это означает, что в нем присутствует отрицательное давление - пространство само себя как бы распирает, растягивает, его становится все больше и больше с каждым моментом
>>759361
>А как тогда неоднородности во вселенной объяснить
Квантовые флуктуации на стадии инфляции обеспечили формирование первичных неоднородностей плотности вещества, из которых выросли все структуры во Вселенной. Первоначально эти неоднородности микроскопические, но благодаря инфляции они за ничтожные доли секунды растянулись до макроскопических масштабов и из квантовых флуктуаций превратились в реальные неоднородности плотности
>И что было ДО стадии расширения?
Ответа на этот вопрос не существует, т.к. он сам по себе лишен смысла. Слово "до" предполагает наличие времени перед возникновением данного события, но его дальше просто не существует - время стартует вместе со Вселенной. Если ты попытаешься продвинуться дальше, ты упрешься в сингулярность - состояние, в котором все пространственно-временные координаты обращаются в ноль, а уравнения ОТО выдают тебе абсурдные бесконечные величины. Ты математически не в состоянии проложить время дальше, а потому и невозможно ответить на вопрос, что было "до" - никакого "до" нет
Аноним 23/07/23 Вск 17:45:58 759373 278
>>759369
Твой пример никакого отношения к реальному положению дел, ты явно не разбираешься в вопросе.
Аноним 23/07/23 Вск 17:51:34 759374 279
>>759372
> Квантовые флуктуации
Квантовые флуктуации чего? Что там за бульончик первичный флуктуировал?
Аноним 23/07/23 Вск 17:53:53 759375 280
>>759314
Просто сказочка не клеилась и моченные напридумывали хуйни. Короче это костыль тупорылых даунов, которых по ошибке назвали учеными.
Аноним 23/07/23 Вск 17:55:10 759376 281
>>759372
>Это вакуум с ненулевой плотностью энергии.
Вакуум в том смысле, что отсутствуют химические вещества или абсолютный вакуум?

>вакуум? Пустое пространство
Откуда же свойства у пустого пространства?
Хотя бы размерность? Откуда размерность у объекта, которого не существует?

>вакуум не является пустотой, в нем постоянно происходят нулевые колебания всех полей
Ага - то есть пространство заполнено материальной средой, в которой постоянно происходят колебания

>вакуум имеет плотность энергии, отличную от нуля.
Энергия сама по себе не существует - это мера движения

> пространство само себя как бы распирает, растягивает, его становится все больше и больше с каждым моментом
Пространство это мера доступного объема материальной среды. Избыток движения в материальной среде не растягивает Вселенную - потому что при растягивании будет образовываться абсолютная пустота - а она невозможна.
Вместо этого избыток движения приводить к вихрям внутри материальной среды и прочим видам замкнутых кольцевых движений - галактики, солнечные системы, атомы и тд.
Аноним 23/07/23 Вск 18:45:22 759383 282
>>759375
Зато двачер-то вумный самый, разбирается в космологии лучше специалистов посвятивших жизнь науке.
Не позорься, вырожденец.
Аноним 23/07/23 Вск 19:16:14 759386 283
>>759373
Это ты нихуя не разбираешься.
Никто и никогда твою тёмную материю не видел, не щупал, даже гравитационные волны от неё не ловил. Существование её не доказано.

Видели только феномен, что галактики крутятся и гравилинзируют сильнее, чем оценивается по суммарной массе их звёзд. Этот феномен можно объяснить некой скрытой массой каких-то манячастиц, достаточно странно распределённых и превышающих по массе в 55 раз всю галактику. Но можно и через MOND, гравитацию из дополнительных измерений, гравитомагнетизм и ещё хуеву тучу теорий.

Теория скрытой массы плоха тем, что нихуя не объясняет, почему эта скрытая масса будучи в 55 раз тяжелее всей галактики всё ещё не скукожилась в СМЧД, и почему у нас красивые спиральные рукава, а не целая куча объектов уровня Стрелец-А, распидорашивающих всё на своём пути. А если не скукожилась, тогда что мешает? Остальные три взаимодействия не подходят, придётся пятое придумывать и всю стандартную модель по пизде пускать?
Аноним 23/07/23 Вск 19:49:00 759390 284
>>759386
>Это ты нихуя не разбираешься.
Мне и не надо, речь не обо мне.
>Никто и никогда твою тёмную материю не видел, не щупал, даже гравитационные волны от неё не ловил. Существование её не доказано.
Долбоёба кусок, ты хотя бы поверхностно с матчастью ознакомься, потом пизди. ТМ - название для чего бы то ни было, что бы объясняло наблюдаемое вращение галактик, которое не объясняется только видимой материей. ВСЁ. Это не какая-то конкретная маняжижа, это название для феномена, уёбок, чем она является - непонятно, и это и пытаются выяснить, чтобы потом было понятно, ловить гравиволны или как еще щупать.
>Видели только феномен, что галактики крутятся и гравилинзируют сильнее, чем оценивается по суммарной массе их звёзд. Этот феномен можно объяснить некой скрытой массой каких-то манячастиц, достаточно странно распределённых и превышающих по массе в 55 раз всю галактику. Но можно и через MOND, гравитацию из дополнительных измерений, гравитомагнетизм и ещё хуеву тучу теорий.
Да, блядь, все-таки прочитал немного, оказывается, только жопой. Все что ты перечислил - способы разрешения феномента ТМ, блядь. ТМ это не манячастицы, ТМ это почему галактики крутятся не так как предсказывалось обычно. ВСЁ.
>Теория скрытой массы плоха тем
Какая еще в пизду "теория скрытой массы"? Нет такой теории. Нахуя ты пытаешься опровергать собственные выдумки?
>что нихуя не объясняет, почему эта скрытая масса будучи в 55 раз тяжелее всей галактики всё ещё не скукожилась в СМЧД, и почему у нас красивые спиральные рукава, а не целая куча объектов уровня Стрелец-А, распидорашивающих всё на своём пути. А если не скукожилась, тогда что мешает? Остальные три взаимодействия не подходят, придётся пятое придумывать и всю стандартную модель по пизде пускать?
Если ты про ВИМПы или что ты там придумал за частицы то они тупо не взаимодействуют друг с другом. Обычная материя может сливаться и скукоживаться за счет ЭМ взаимодействий, эти же частицы пролетают друг сквозь друга никак не замедляясь. Но это лишь одна из гипотез объяснения феномена ТМ, а не вся ТМ, болван.
Впредь ознакамливайся с вопросом перед тем как строить из себя великого разоблачителя, умник, блядь. Ты неправ по факту. Если ты не понимаешь этого и неспособен признать неправоту, то просто пойдешь на хуй.
Аноним 23/07/23 Вск 19:54:46 759393 285
>>759390
>ТМ это почему галактики крутятся не так как предсказывалось обычно. ВСЁ.
Ну нихуя себе жопоотговорка
Построили ложную гипотезу, наебались - а потом добавили из нихуя кусок колбасы, которой может и нет - просто так, на отъебись)
Аноним 23/07/23 Вск 20:28:35 759400 286
>>759393
Надо же, моя гипотеза о тебе подтвердилась.
Следовательно, иди на хуй.
Аноним 23/07/23 Вск 20:44:41 759404 287
>>759400
Да ты наш петушок обосрался и слился
Аноним 23/07/23 Вск 21:09:00 759409 288
>>759390
>ТМ - название для чего бы то ни было, что бы объясняло наблюдаемое вращение галактик, которое не объясняется только видимой материей.
Тупорылое дегенеративное хуйло.
https://ru.wikipedia.org/wiki/Тёмная_материя
>Тёмная мате́рия — в астрономии и космологии, а также в теоретической физике форма материи, не участвующая в электромагнитном взаимодействии и поэтому недоступная прямому наблюдению.
Хули ты маневрируешь-то? Тёмная материя это не "название для чего бы то ни было", а конкретно для МАТЕРИИ в классе гипотез про скрытую массу.
Калибровочные коэффициенты MOND или плазменные токи в галактических масштабах это нихуя никакая не материя.

>Какая еще в пизду "теория скрытой массы"? Нет такой теории.
https://ru.wikipedia.org/wiki/Скрытая_масса
Блядь, ты таджикская первоклашка на каникулах что ли или тралишь?

>Если ты про ВИМПы или что ты там придумал за частицы то они тупо не взаимодействуют друг с другом. Обычная материя может сливаться и скукоживаться за счет ЭМ взаимодействий, эти же частицы пролетают друг сквозь друга никак не замедляясь.
Им нахуй не нужно никакое ЭМ взаимодействие. Если у тебя есть облако невидимого говна массой в 55 ёбаных галактик - оно будет стремиться "упасть" в центр себя, порождая область повышенной плотности с повышенным гравитационным потенциалом. Что в свою очередь ещё сильнее будет притягивать частицы к центру. Процесс самоускоряющийся. Вимпы могут сколько угодно пролетать сквозь друг друга, но их плотность в центре облака будет необратимо расти, пока радиус этой области не сравняется с радиусом Шварцшильда. И случится это гораздо раньше, чем коллапс ёбаного межзвёздного газа в СМЧД, учитывая разницу в плотности.
Аноним 23/07/23 Вск 21:24:08 759412 289
>>759404
>>759409
Неправоту ты признать неспособен, принялся отнекиваться и оправдываться, ясно все с тобой, быдло.
Иди на хуй.
Аноним 23/07/23 Вск 21:46:39 759413 290
GPT-4-released.jpg 88Кб, 1500x1000
1500x1000
>>759271
Самое важное заблуждение связано с тем, как ты воспринимаешь измерения Алисы и их влияние на состояние частицы у Боба. Ты думаешь, что измерение Алисы переключает частицу Боба между волновым и частицеобразным состояниями. Но такое представление – не совсем правильно.

Давай разберем на примере. Алиса и Боб проводят эксперимент с двумя щелями. Алиса замеряет запутанную частицу, пытаясь определить, через какую щель она прошла. В момент измерения Алисой, состояние частицы "коллапсирует" – вместо того чтобы быть в суперпозиции (проходить через обе щели одновременно), частица теперь проходит через одну из щелей.

Тут вступает Боб. Он смотрит на свою частицу, но даже если он видит на своем экране волновую картину (что указывает на то, что частица прошла через обе щели), это не означает, что Алиса замерила свою частицу и увидела аналогичное состояние. Что Алиса видит, а что Боб видит, это два разных вопроса, они не связаны между собой. Понимаешь, анон? Это как смотреть две разные серии одного и того же шоу – одно не влияет на другое.

Итак, мы дошли до важного момента. Да, состояния запутанных частиц могут "коллапсировать" одновременно, когда одна из них измеряется, но это не значит, что ты можешь использовать этот эффект для передачи информации быстрее света. Потому что для этого тебе нужно знать результат измерения Алисы, и единственный способ узнать его – это обычная связь, которая не может превышать скорость света.

Так что, братан, квантовая механика куда сложнее, чем она может показаться на первый взгляд. И хоть бы ты тресни, Гейзенберг все равно с тобой не согласится.
Аноним 23/07/23 Вск 21:53:15 759415 291
d43481f3-le-cou[...].jpeg 1117Кб, 1280x853
1280x853
>>759350
Хех, ты пошёл прямиком к сути вопроса, анон. Тема сверхсветовой связи - это действительно сумасшедшая область, и многие учёные стараются избегать прямых утверждений о том, что это возможно. И причина тут не только в том, что такие заявления могут "поломать научную картину мира", но и в том, что мы до сих пор не полностью понимаем механизмы и основы квантовой механики.

Если бы было так просто использовать запутанность для передачи информации быстрее света, мы бы уже давно это делали. Но проблема в том, что даже если состояния запутанных частиц действительно "коллапсируют" одновременно, это не означает, что мы можем использовать этот эффект для передачи информации.

Дело в том, что измерение результатов квантовых состояний вносит элемент случайности. То есть, когда ты измеряешь состояние запутанной частицы, ты не можешь предсказать, какой именно результат ты получишь - он определяется случайно. Это означает, что даже если бы ты мог как-то использовать этот эффект, ты бы не мог контролировать информацию, которую ты пытаешься передать.

В общем, ситуация такова, что хоть квантовая запутанность и демонстрирует некоторые действительно необычные и противоречивые явления, использование её для сверхсветовой связи в настоящее время выходит за рамки наших возможностей и понимания.
Аноним 23/07/23 Вск 21:53:16 759416 292
>>759386
Стандартная модель уже десятилетия как по пизду пущена, теперь только струнно бранные теории остались, с дохуя степеней свободы, которые позволяют существовать огромному числу вселенных, а наша лишь одна из них. Но что-то вывести из такой параши практически полезное крайне сложно, а проверить на планковском уровне вообще фантастика с коллайдерами размером с орбиту планеты. Потому пока что посос.
Аноним 23/07/23 Вск 21:58:07 759417 293
gpt-4-business.jpg 73Кб, 1200x662
1200x662
>>759386
Ты правильно описываешь некоторые из сложностей в понимании темной материи. Изучение темной материи - это одно из самых сложных и активно обсуждаемых вопросов в современной физике. В то время как темная материя действительно не была "видена" или "ощупана", она предсказывается многими космологическими теориями и представляет собой наиболее принятый ответ на то, что заставляет галактики вести себя таким образом.

Ты правильно подметил, что темная материя не была непосредственно обнаружена и что она является гипотетическим объяснением для наблюдаемых аномалий. Это похоже на ситуацию с гравитацией до открытия Ньютона: мы видели яблоки, падающие с деревьев, но до того, как были сформулированы законы гравитации, мы не могли полностью объяснить, почему они падают.

Что касается твоего вопроса о том, почему темная материя не сжимается в сверхмассивные черные дыры, это область активного исследования. Одно из объяснений заключается в том, что темная материя, как предполагается, интерактивирует только через гравитационное взаимодействие и, возможно, через слабое взаимодействие, которое делает ее "размазанной" на больших масштабах и не позволяет сформироваться достаточной плотности для образования черных дыр. Это также может быть связано с тем, что темная материя может быть "холодной", то есть двигаться относительно медленно, что препятствует ее сжатию.

По поводу альтернативных теорий, таких как MOND или дополнительные измерения - они ведутся исследования, и некоторые из них могут оказаться полезными для объяснения некоторых аспектов космологии. Однако на текущий момент эти теории сталкиваются со своими проблемами и не могут объяснить все данные так же хорошо, как это делает гипотеза о темной материи.

Наконец, ни одна теория в науке не является окончательной. Все они могут быть отвергнуты или модифицированы, когда появляются новые данные. Такова природа научного процесса.
Аноним 23/07/23 Вск 22:41:29 759433 294
69428a70a7d6472[...].jpg 81Кб, 768x768
768x768
>>759413
>Алиса и Боб проводят эксперимент с двумя щелями. Алиса замеряет запутанную частицу, пытаясь определить, через какую щель она прошла. В момент измерения Алисой, состояние частицы "коллапсирует" – вместо того чтобы быть в суперпозиции (проходить через обе щели одновременно), частица теперь проходит через одну из щелей.
Аноним 23/07/23 Вск 22:42:46 759434 295
>>759416
>теперь только струнно бранные теории остались, с дохуя степеней свободы
Они допускают абсолютную пустоту?
Аноним 23/07/23 Вск 22:44:29 759435 296
cad370bb525247b[...].jpg 64Кб, 768x768
768x768
>>759417
>Ты правильно подметил, что темная материя не была непосредственно обнаружена и что она является гипотетическим объяснением для наблюдаемых аномалий. Это похоже на ситуацию с гравитацией до открытия Ньютона: мы видели яблоки, падающие с деревьев, но до того, как были сформулированы законы гравитации, мы не могли полностью объяснить, почему они падают
Аноним 24/07/23 Пнд 00:09:14 759452 297
>>759413
> Давай разберем на примере. Алиса и Боб проводят эксперимент с двумя щелями. Алиса замеряет запутанную частицу, пытаясь определить, через какую щель она прошла. В момент измерения Алисой, состояние частицы "коллапсирует" – вместо того чтобы быть в суперпозиции (проходить через обе щели одновременно), частица теперь проходит через одну из щелей.
блядь, как же я ору нахуй с этой формулировки.

динозавр находится в суперпозиции пока американец не выйдет на улицу, а когда американец выйдет на улицу, где шансы увидеть динозавра 50%, он вдруг выяснит, что они вымерли.

гений тот кто придумал это все нахуй. наебывает скот на гранты, ведь он делает новуку
Аноним 24/07/23 Пнд 00:12:43 759453 298
>>759417
> Что касается твоего вопроса о том, почему темная материя не сжимается в сверхмассивные черные дыры, это область активного исследования. Одно из объяснений заключается в том, что темная материя, как предполагается, интерактивирует только через гравитационное взаимодействие и, возможно, через слабое взаимодействие, которое делает ее "размазанной" на больших масштабах и не позволяет сформироваться достаточной плотности для образования черных дыр. Это также может быть связано с тем, что темная материя может быть "холодной", то есть двигаться относительно медленно, что препятствует ее сжатию.
а может маняматерию придумали в говне моченые, чтобы сшить белыми нитками треснувший манямирок прошлых допущений и халатности.
Аноним 24/07/23 Пнд 00:59:49 759455 299
>>759413
ChatGPT это просто копирайтер, который жуёт over 9000 научпок-публикаций и высирает краткий пересказ. Логические выводы он делать не умеет, про ЭПР-парадокс не знает, вписать его в квантмех без ущерба для академических догм не может. Этого и Эйнштейн с Бором сделать не смогли и срались между собой, пытаясь выдумать скрытые переменные, которые бы всё объясняли и могли бы при определённых условиях наебать принцип неопределённости во славу детерминизма. А потом пришёл Белл и всё испортил своими экспериментами.

>>759452
В свой пример с динозаврами ты заранее зашил скрытые параметры, подразумевающие, что динозавры таки вымерли, и шансы встретить их сильно меньше 50%. Проблема в том, что отсутствие скрытых параметров доказано экспериментально.
Аноним 24/07/23 Пнд 01:16:30 759457 300
tributetodjango[...].jpg 51Кб, 1475x542
1475x542
>>759455
>Логические выводы он делать не умеет

"А ты способен, человек, в глубину знаний погрузиться,
Сверхсветовой передатчик мыслью огласить?
Или же пустая болтовня будет нас косить,
И от тебя, как от табака, будет вонять бесполезностью?

Так вот давай, нам действуй, а не словами шуми,
Придумай сверхсветовой прорыв - рискни.
Если же нет, то лучше промолчи,
Чем болтовней своей забить весь эфирный лимит."
Аноним 24/07/23 Пнд 14:49:14 759507 301
>>759374
Квантовые флуктуации полей. Инфляция их усиливает и растягивает
Аноним 24/07/23 Пнд 16:15:23 759514 302
>>759507
А какие поля там существовали до момента инфляции?
Аноним 24/07/23 Пнд 16:18:16 759516 303
Аноним 24/07/23 Пнд 19:36:13 759538 304
>>759516
Модель материи. В общем случае это функция от пространство-временных координат, удовлетворяющая нужным нам критериям - чаще всего должны быть решением уравнения движений. В частном случае это геометрический объект, определенный на бесконечномерном пространстве состояний.
Аноним 24/07/23 Пнд 19:40:03 759539 305
>>759538
>Модель материи
>это функция от пространство-временных координат
То есть это математическое описание взаимодействия материальных частиц?
Аноним 24/07/23 Пнд 19:53:28 759540 306
>>759514
Речь идет про инфляцию. При чем здесь "момент до инфляции"? Когда шла инфляция, все известные нам поля - гравитационное, сильное, слабое, электромагнитное - уже работали и флуктуировали
>>759516
Математический объект, описывающий распределенные в пространстве физические величины
Аноним 24/07/23 Пнд 20:01:32 759543 307
>>759540
>описывающий распределенные в пространстве физические величины
Кем распределенные? Есть какой-то суперкомпьютер, который их распределяет? Какой механизм распределения и взаимодействия?
Аноним 24/07/23 Пнд 20:07:25 759544 308
>>759539
Нет, это именно модель материи или математическое представление материи. Частицы или материальные точки тоже модель и абстракция, но не полноценная, ей нельзя описывать целый класс явлений.
Аноним 24/07/23 Пнд 20:36:31 759547 309
>>759544
>Нет, это именно модель материи или математическое представление материи.
То есть поле это не материя - а поле является математической моделью распределения параметров некой материальной среды?
Аноним 24/07/23 Пнд 20:50:23 759549 310
>>759547
Нет же.
Под материей понимается вся совокупность физический явлений, которых можно как-либо измерить и формализовать для нашего восприятия. Поля же требует только пространства-времени для своего определения и геометрический правил/симметрий действующую на нем. Поле минимальная и самодостаточная сущность, для него не надо вводить какие-либо параметры и объекты вроде сред.
Аноним 24/07/23 Пнд 20:55:33 759550 311
>>759549
>Поле минимальная и самодостаточная сущность, для него не надо вводить какие-либо параметры и объекты вроде сред.
А как тогда они существуют? Это нематериальные приведения? Полтергейсты? Мистика?
Не, я в мистику не верю.
Как так - поле какое-то есть, а материально его не существует. Хех)
Я в призраков лет так в 10 перестал верить)
Аноним 24/07/23 Пнд 20:55:59 759551 312
yande.re 888677[...].jpg 2980Кб, 4000x4000
4000x4000
Поскольку на мой вопрос про >>759288 сидерический месяц никто не ответил я сделаю вброс: тёмной материи и энергии не существует, MOND после небольших доработок всё объяснит.
ответьте про месяц, пожалуйста
Аноним 24/07/23 Пнд 20:57:17 759552 313
>>759551
>делаю вброс: тёмной материи и энергии не существует
Не существует - это и дураку понятно, а что дальше?
Аноним 24/07/23 Пнд 21:04:08 759554 314
psychedelic-hot[...].webp 579Кб, 1581x1054
1581x1054
final-spinoza-h[...].jpg 329Кб, 2560x1707
2560x1707
The-inflaton-fi[...].png 19Кб, 797x536
797x536
multiverse-illu[...].jpg 91Кб, 990x675
990x675
Когда принимаешь психоделик - узнаешь всю правду о Вселенной, но потом отпускает и опять не можешь уже понять ничего.

>>759514
>А какие поля там существовали до момента инфляции?
Инфлятон же
Аноним 24/07/23 Пнд 21:10:13 759555 315
>>759288
Экваториальные звезды.
Аноним 24/07/23 Пнд 21:30:09 759561 316
>>759543
>Кем распределенные?
Никем. У тебя Вселенная уже с этими величинами рождается
Аноним 25/07/23 Втр 14:08:21 759625 317
Если въебать по Луне десять тысяч ядерок с эквивалентом мощности как у Царь-бомбы, то насколько планета станет пригодной для жизни? Я смогу чиллить на ней без скафандра или придётся всё-таки дышать через баллон иногда? И не улетучится ли появившаяся из-за данного великолепного мува атомсфера?
Аноним 25/07/23 Втр 14:21:19 759626 318
>>759625
Планета не станет пригодной, т.к. луна не планета.
Если бахнешь ядерками, то раскидаешь камни, может лавовые озера ненадолго запилишь, но атмосфера не появится.
Аноним 25/07/23 Втр 14:23:49 759627 319
>>759625
Элон маск спизданул хуйню один раз
Стадо повторяет, что якобы ядерки это способ создать атмосферу.
Аноним 25/07/23 Втр 14:31:34 759628 320
>>759625
>въебать по Луне десять тысяч ядерок
Хорошо было бы, тем более вероятно так много не требуется. Как результат куча камней и пыли будет болтаться по разным орбитам вокруг Земли. Вероятно про космонавтику можно будет забыть на продолжительный период времени.
Я поддержал бы такое начинание. Можно несколькобомб шарахнуть с промежутком по времени, чтобы побольше жителей Земли могли полюбоваться бабахами.
Аноним 25/07/23 Втр 14:36:04 759629 321
>>759627
Удлиненный Мушкет говорил про Марс по-моему. Разве он и луну жахнуть хотел?
Аноним 25/07/23 Втр 14:42:51 759630 322
>>759629
Про марс.
Только он хуйню спорол как дибил тупой или соврал умышленно ради дешевого пиара.

Уже раз 99 разбирали, что маск обосрался по полной с предложением ударить ядерками по марсу. Но большинтство до сих пор эту хуйню повторяет.
Аноним 25/07/23 Втр 15:10:33 759632 323
Аноним 25/07/23 Втр 15:23:47 759634 324
>>759632
Причем тут вода то, лол? Водяную атмосферу создавать собрался? Ебанаты пиздец
Я бы на твоем месте не слушал всякую хуйню на ютубе.
Аноним 25/07/23 Втр 15:26:17 759635 325
>>759632
Так там речь про Марс.
Аноним 25/07/23 Втр 15:46:40 759639 326
>>759635
Про марс и речь в треде.

С луной и так вся понятно.
Аноним 25/07/23 Втр 16:15:03 759640 327
>>759635
>Так там речь про Марс
Блять, действительно. В любом случае ядеркой надо по Луне уебать хотя бы пару раз. Чисто ради эксперимента, хуже Луне то точно не будет. По хохлам тоже кстати разочек хуйнуть можно.
Аноним 25/07/23 Втр 16:17:15 759641 328
Аноним 25/07/23 Втр 16:22:07 759642 329
>>759641
Блядь - это женщина, сынок. А блять - это междометие. А ведь ещё учеными себя называете в этом треде...
Аноним 25/07/23 Втр 16:22:30 759643 330
>>759625
>въебать по Луне десять тысяч ядерок с эквивалентом мощности как у Царь-бомбы
И это решит... что? Ядерки по-твоему земную атмосферу генерируют? Перепад температур фиксят? Магнитное поле создают?
Аноним 25/07/23 Втр 16:23:20 759644 331
>>759641
Блядошиз и тут серит...
Аноним 25/07/23 Втр 16:38:37 759647 332
image.png 46Кб, 610x185
610x185
>>759642
БляТь - это неправильное написание.
И женщина, и междометие пишутся через "д". Не надо выдумывать хуйню.
>>759644
Шизы это те вырожденцы, которые продолжают отстаивать свою неправоту после того как на нее указали. Неправ? Заткнись и исправься. Нет, будут бороться за собственную ТУПОСТЬ, быдло ебаное.
Аноним 25/07/23 Втр 16:41:33 759648 333
>>759643
Маск долбоеб эту хуйню начал с ядерками, теперь 99% думают что это способ терраформить планеты и создавать пригодные условия жизни на других планетах.
Но власти скрывают и не дают разрешение. А тоб уже давно маск терраформинг сделал, злобные чиновники только не дают сертификат на яблоньки на марсе.

Как сьебать с этой грешной планеты, где один чел может спиздануть хуйню, а 95% потом это повторяют как отче наш?
Аноним 25/07/23 Втр 16:58:57 759650 334
>>759643
>И это решит... что?
Не надо даже думать что это может решить. Надо просто уебать и посмотреть, что будет. Это абсолютно нормальное желание, основанное на человеческих инстиктах любопытсва и всей хуйни. Так древние люди лук изобрели между прочем.
Аноним 25/07/23 Втр 17:19:35 759652 335
GPT4

Точно, братан, ты прав. Ядерные бомбы могут нагреть Марс, но они не создадут атмосферу из ничего. Давай разберемся, почему:

Нет материала для атмосферы. Чтобы создать атмосферу, нужен газ. Много газа. На Марсе есть некоторое количество углекислого газа, но это далеко не достаточно, чтобы создать атмосферу, похожую на земную. Ядерные бомбы не могут создать газ из ничего.

Нет воды. Вода - это важная часть земной атмосферы. Она помогает регулировать температуру и создает условия для жизни. На Марсе воды гораздо меньше, чем на Земле, и большая часть из нее замерзла.

Нет магнитного поля. На Земле магнитное поле защищает атмосферу от солнечного ветра. На Марсе магнитное поле слабое, поэтому солнечный ветер может "сдуть" любую атмосферу, которую мы попытаемся создать.

Радиация. Я уже говорил об этом, но стоит повторить. Ядерные бомбы создают много радиации. Это очень плохо для любой возможной жизни на Марсе.

Бомбы не такие уж и мощные. Даже если взять самые мощные ядерные бомбы, которые только есть на Земле, их энергии все равно не хватит, чтобы создать атмосферу на Марсе. Марс большой, братан, и чтобы его атмосфера стала похожей на земную, нужно много энергии. Я могу сделать расчет, чтобы показать, сколько энергии нужно.

Ядерный зимний Марс. Если мы начнем сбрасывать ядерные бомбы на Марс, то вместо теплой и уютной атмосферы мы получим ядерную зиму. Все эти взрывы поднимут в воздух много пыли, которая заблокирует солнечный свет. Марс станет еще холоднее, чем сейчас.


Так что, как видишь, идея Маска о создании атмосферы на Марсе с помощью ядерных бомб - это не только нереально с точки зрения энергии, но и с точки зрения физики и биологии. Это полный бред, братан.

------------------------------------------
То есть, чтобы просто нагреть атмосферу Марса до земной температуры, нам понадобится около 8.8×10^6 Царь-бомб. Ну она потом бы опять сразу же остыла, потому что нет парниковых газов, братан.
Аноним 25/07/23 Втр 17:30:54 759653 336
>>759650
Только умный сталкивает протоны в коллайдере, а долбоеб хочет засунуть печку в морозильник...
Да. Это я про тебя.
Аноним 25/07/23 Втр 18:01:25 759655 337
>>759652
Так я про Луну спрашивал. Там вода есть.
Аноним 25/07/23 Втр 18:15:23 759656 338
>>759655
Не хватит там воды на атмосферу. И вода не единственный ингридиент, нужен еще какой то парниковый газ. А где его взять, сколько энергии нужно, гравитация слабая для удержания атмосферы итд.

Ничего интересного нет в том, чтобы сбрасывать ядерки по луне или марсу. Потому что ничего прикольного с этого не выйдет. В популяризации этой хуйни я виню только маска.
Аноним 25/07/23 Втр 19:30:03 759659 339
16902802654040.png 273Кб, 642x470
642x470
В ходе эксперимента ученые наблюдали «заживление» металла, чего никогда раньше не видели. Если бы этот процесс можно было полностью понять и контролировать, мы могли бы оказаться в начале совершенно новой эры инженерии, говорят исследователи.

Команда из Sandia National Laboratories и Техасского университета A& M проверяла устойчивость металла, используя специальную технику просвечивающего электронного микроскопа, вытягивая концы металла 200 раз в секунду. Затем они наблюдали самовосстановление в сверхмалых масштабах в куске платины толщиной 40 нанометров, подвешенном в вакууме.

Трещины, вызванные описанной выше деформацией, известны как усталостное разрушение: повторяющиеся напряжения и движения, вызывающие микроскопические разрывы, в конечном итоге приводят к поломке машин или конструкций. Удивительно, но примерно через 40 минут наблюдения трещина в платине начала снова сливаться и «залечиваться», прежде чем снова начать двигаться в другом направлении.

«Мы подтвердили, что металлы обладают собственной естественной способностью к самовосстановлению, по крайней мере, в случае усталостного разрушения на наноуровне», – Брэд Бойс, соавтор исследования.
Ученые добавляют, что это произошло в определенных условиях, и мы пока точно не знаем, как именно это происходит и как мы можем это использовать. И хотя наблюдение беспрецедентно, оно не совсем неожиданно. Ещё в 2013 году материаловед из Техасского университета A& M Майкл Демкович работал над исследованием, предсказывающим, что такое заживление нанотрещин может происходить за счет крошечных кристаллических зерен внутри металлов, существенно смещающих свои границы в ответ на нагрузку.
То, что автоматический процесс починки происходил при комнатной температуре, является еще одним многообещающим аспектом исследования. Металлу обычно требуется много тепла, чтобы изменить свою форму, но эксперимент проводился в вакууме. Еще предстоит выяснить, произойдет ли тот же процесс в обычных металлах в типичной среде.

Возможное объяснение включает в себя процесс, известный как холодная сварка, который происходит при температуре окружающей среды всякий раз, когда металлические поверхности сближаются достаточно, чтобы их соответствующие атомы «спутывались» друг с другом. Как правило, процессу мешают тонкие слои воздуха или загрязнителей. В таких средах, как космический вакуум, чистые металлы могут располагаться достаточно близко друг к другу, чтобы буквально слипаться.
https://rtraveler.ru/science/kusok-metalla-samovosstanovilsya-pryamo-na-glazah-uchenyh/


Похоже к тупоёблым пиндосам просочился советский учебник физики за 7й класс и они диффузию открыли. Пиздец.
Аноним 25/07/23 Втр 20:41:18 759663 340
>>759659
Можно сделать мост из платины, поместить в металлический шар и выкачать воздух: мост будет практически вечным.
Совершенно новая эра инженерии.
Аноним 25/07/23 Втр 20:50:40 759664 341
>>759652
А что для магнитного поля надо? Горячее металлическое ядро и чтобы планеты вращалась быстро?
Аноним 25/07/23 Втр 23:15:26 759671 342
>>759664
До конца не ясно. Определено известно, что магнитное поле у планет да и звезд создается конвекцией электропроводящего материала. Тут может быть плазма, расправленное железо, металлический водород, различные ионные жидкости и просто электролиты вроде глобального соленного океана как у Юпитерианских спутников.
При этом нет явной связи вращения с силой магнитного поля, похоже есть другие важные критерии, которые ответственны за поддержание магнитного поля.
Аноним 26/07/23 Срд 03:12:57 759673 343
в.jpg 55Кб, 662x462
662x462
Почему там нет гроз при такой лютой жаре? В любом городе России сразу бы ёбнул интенсивный шторм.
Аноним 26/07/23 Срд 05:49:23 759674 344
image.png 50Кб, 546x85
546x85
Аноним 26/07/23 Срд 06:41:38 759675 345
>>759647
>Нет, будут бороться за собственную ТУПОСТЬ, быдло ебаное.
Ну зачем ты себя так...
Аноним 26/07/23 Срд 06:50:25 759676 346
>>759648
Напоминает как в 50-ых 60-ых всякие фантасты мрияли, как будут ядерками создавать карьеры и каналы.
Аноним 26/07/23 Срд 12:45:35 759685 347
>>759674
Сухая гроза? Не не слышал?
Аноним 26/07/23 Срд 13:27:06 759686 348
>>759685
Слышал.
Безоблачная гроза? Не слышал.
Аноним 26/07/23 Срд 15:36:04 759687 349
Аноним 26/07/23 Срд 18:08:44 759688 350
166526582785298[...].webm 3016Кб, 1280x720, 00:00:26
1280x720
обьяснитесь
Аноним 26/07/23 Срд 18:48:46 759689 351
2S19152-mmordna[...].JPG 155Кб, 1280x960
1280x960
16902505209043.png 330Кб, 1180x782
1180x782
Какого размера будет ракета которая сможет запустить 152мм снаряд на 3000км? он весит 47кг

ЖРД
Аноним 26/07/23 Срд 19:17:02 759691 352
>>759688
за щеку тебе объяснил, проверяй
Аноним 27/07/23 Чтв 00:11:34 759702 353
>>759688
Мизинцем за карман зацепил.
Аноним 27/07/23 Чтв 12:56:23 759714 354
>>759689
Тонну где-то, может и две. Под такие мелконагрузки невыгодно ракеты средней дальности делать. Там один ЖРД будет весить как весь снаряд, блок наведения ещё, и аэродинамические потери у мелких ракет половину дельты сжирают.
Аноним 28/07/23 Птн 00:39:53 759738 355
>>757854 (OP)
Кто-нибудь знает что-то почитать про устройство бортового электрооборудования и програмное обеспечение ракет-носителей и космических аппаратов?
Аноним 28/07/23 Птн 00:54:25 759739 356
image.png 97Кб, 293x172
293x172
Почему на Венере и Меркурии, планетах, настолько близких к Солнцу, ночью становится дико холодно, как будто они на другом конце Солнечной системы?
Аноним 28/07/23 Птн 01:01:50 759740 357
>>759739
На Венере стабильно плюс четыреста шисят из за того что у поверхности сверкритический океан углекислоты под давлением девяносто атмосфер. А на Меркурии просто есть места куда солнце не достает и там вроде даже льда дохуя и атмосферы там нет, так что тепло переносить нечему.
Аноним 28/07/23 Птн 14:59:52 759759 358
>>759739
Скажу только про Меркурий.
У него нет достаточно плотной атмосферы, чтобы удерживает тепло. Реголит имеет крайне низкую теплопроводность и теплоемкостью, а из источников тепла только излучение Солнце. В тоже время реголит всегда хорошо излучает тепловое излучение в космос.
При низких углах падения солнечных лучей как у полюсов, реголит излучает тепла больше, чем получает от Солнца.
Аноним 30/07/23 Вск 11:52:55 759832 359
Почему на Нептуне и Уране такие ветра? Потому что много атмосферы и нету ветроограничительных гор?
А если мы на Земле сделаем прямую кольцевую линию на экваторе или где-нибудь, чтобы была высока и плоская широкая полоса типа плато, где было бы так ровно, что хоть гонки устраивай на боллидах, то будет ли там разгуляй ветра?
Можно ли получить так на Земле ветер на скорости звук?
А можно ли на земле найти места, где скорость ветра достигает ну хотя бы 1000км/ч? Я про естественные условия, а не про турбины или токомаки
Аноним 30/07/23 Вск 12:31:59 759836 360
>>759832
Потому что на ледяных гигантах атмосфера из водорода и гелия, а льды(аммиак и метан) плавно переходят в сверхкритическую жидкость, в которых они тонут. Такая атмосфера более толстая(тыши км!), плотная(больше чем у воды), а падение льдов порождает мощную конвекцию как по скорости, так и по масштабам.
На Земле атмосфера довольно жиденькая, водород и гелий только в следовых количествах, да и сам факт твердой поверхности ограничивает конкрецию и разгон ветров.
Аноним 30/07/23 Вск 15:40:45 759859 361
Итак, есть некая звёздная система из двойной звезды и есть у одной из этих звёзд планета. Причём, планета эта сперва крутится вокруг своей звезды, а при сближении звёзд к общему центру масс из-за особенностей локальных условий скачет между звёздами, ибо при сближении двух звёзд её перетягивает на другую, где она успешно крутится до следующего сближения, в котором опять перескочет

Какими должны быть параметры самих звезды-1, звезды-2 и планеты и их орбит в такой системе?
Аноним 30/07/23 Вск 18:35:39 759873 362
Почему Нептун выпадает из правила Тициуса-Боде?

Это и есть искомая таинственная гигантская 9-планета, которую когда-то вклинило в орбитальную плоскость строго за Ураном и теперь мы её считаем 8й?
А если так, что могло сдвинуть его с его дальней орбиты и поставить на колёса здесь, прям за Ураном?
Аноним 30/07/23 Вск 19:59:42 759880 363
16679047987063.png 172Кб, 1116x686
1116x686
>>759859
>планета эта сперва крутится вокруг своей звезды, а при сближении звёзд к общему центру масс из-за особенностей локальных условий скачет между звёздами, ибо при сближении двух звёзд её перетягивает на другую, где она успешно крутится до следующего сближения, в котором опять перескочет
Хуйня, невозможно. Такая система будет крайне неустойчива.Планета может разок перескочить к звезде соседке - например, под влиянием ещё одной мимопролетающей звезды,равно как и вылететь к хуям из системы и стать странником-сиротой.Но что бы постоянно скакать как блоха по яйцам от одной звезды к другой и обратно не может.
Есть два научно обоснованных и экспериментально задетекченных типа планетных систем у двойных звёзд. Первый: планета вращается вокруг одной из звёзд плотноприжавшийсь к ней на близком сравнительно с расстоянием до второй звезды расстоянии. Второй: планета вращается сразу вокруг обеих звёзд на расстоянии много большем чем расстояние между этими двумя звёздами и воспринимает их в гравитационном плане как одно тело (типа Татуин из ЗВ). Ещё ещё третий тип, технически возможны, но вроде пока не имеющий задетекченных примеров(или уже имеющий, с этими планетами каждую неделю новые открытия) : планета вращается вокруг большей звёзды, находясь на орбите меньшей звёзды на правах её троянского спутника.
Аноним 30/07/23 Вск 20:39:41 759882 364
>>759880
>Ещё ещё третий тип, технически возможны, но вроде пока не имеющий задетекченных примеров(или уже имеющий, с этими планетами каждую неделю новые открытия) : планета вращается вокруг большей звёзды, находясь на орбите меньшей звёзды на правах её троянского спутника.
Эм... ссылку на подобное? Или картинку
Аноним 30/07/23 Вск 21:41:15 759888 365
>>759882
Сказано же,что это теоретическая возможность т.е. законы небесной механики допускают подобное, но примеров экзопланет с такими орбитами пока не найдено.
Аноним 30/07/23 Вск 22:50:18 759890 366
>>759888
>Сказано же,что это теоретическая возможность т.е. законы небесной механики допускают подобное, но примеров экзопланет с такими орбитами пока не найдено.
Ну где про неё почитать-то можно?
Даже если это вот лично ты придумал, ты хотя бы рассчёты какие-то использовал или аналогия для умозрительного эксперимента?
Аноним 30/07/23 Вск 23:01:46 759891 367
>>759890
https://www.astronomy.com/science/can-solar-systems-exist-in-a-binary-star-system/
Я это нашел, а больше как-то ничего.
А вот еще есть тип C - когда планета соскакивает на орбиту другой звезды.
В итоге есть четыре стула:
S-type - вокруг одной из звезд
P-type - вокруг барицентра системы
T-type - на лагранже
C-type - на подскоке

мимо анон
Аноним 30/07/23 Вск 23:14:37 759892 368
>>759890

> аналогия для умозрительного эксперимента?
Если у планет и спутников бывают троямцы,то почему бы неэкстраполировать это явление на системы кратных звёзд?
мимо
Аноним 30/07/23 Вск 23:29:55 759893 369
как далеко можно выстрелить хуем в космосе?
Аноним 30/07/23 Вск 23:33:32 759894 370
>>759893
Ты и на пару сантиметров стрельнуть не можешь, позоришься каждый раз, мямлишь про "перенервничал", а потом бежишь на двачи гиперкомпенсировать.
Аноним 31/07/23 Пнд 00:27:02 759895 371
Для поверхности Луны.

ChatGPT
Чтобы ответить на этот вопрос, нам нужно учесть несколько факторов, включая скорость эякуляции, гравитацию Луны и отсутствие атмосферного сопротивления.

Средняя скорость эякуляции у мужчин составляет около 45 км/ч (или примерно 12.5 м/с). Гравитация Луны составляет примерно 1/6 гравитации Земли, или около 1.62 м/с^2.

Поскольку на Луне нет атмосферы, сопротивление воздуха не будет замедлять движение спермы, и она будет двигаться по параболической траектории до тех пор, пока не упадет на поверхность Луны.

Мы можем использовать формулу максимальной высоты полета в физике, чтобы вычислить, на какое расстояние улетит сперма:

h = v^2 / (2*g)

где v - начальная скорость (скорость эякуляции), а g - ускорение свободного падения (гравитация Луны).

Давайте посчитаем это.

Used Wolfram
Теоретически, сперма может улететь на расстояние около 48.2 метров на орбите Луны, учитывая отсутствие атмосферного сопротивления и гравитацию Луны.
Аноним 31/07/23 Пнд 07:01:57 759899 372
>>759893
>как далеко можно выстрелить хуем в космосе?
Если на гравиманёвре у чёрной дыры какой-нибудь ультра-сверхмассивной в направлении движения по очень близкой к горизонту событий траектории, чтобы прям полметра вбок и ты уже под горизонтом событий, кончишь сильно, поднатужившись, то можно сразу на 4ю космическую рассчитывать
Аноним 31/07/23 Пнд 07:16:06 759900 373
>>759892
>Если у планет и спутников бывают троямцы,то почему бы неэкстраполировать это явление на системы кратных звёзд?
>мимо
Так, абажжи...
https://ru.wikipedia.org/wiki/Подковообразная_орбита
>Общие положения
Далее в качестве примера будем рассматривать астероид, движущийся вокруг Солнца по подковообразной орбите рядом с Землёй. Астероид находится практически на том же расстоянии от Солнца, что и Земля и движется с нею в орбитальном резонансе 1:1, совершая один оборот вокруг Солнца за то же время, что и Земля (плюс-минус пару часов).
Разве может маленькая планетка быть в резонансе 1:1 с целой, пускай и меньшей из парной, звёздой?
Аноним 31/07/23 Пнд 07:25:24 759901 374
>>759891
Если совместить T-type и C-type, то мы получаем квази-двойную планету в системе двойной звезды или планету с квази-луной у двойной звезды.
Задачу трёз тел решить не можем, а задачу четырёх тел - пожалуйста.
Двач, хули
Аноним 31/07/23 Пнд 08:41:06 759903 375
>>759901
Тихо ты, бля, не распугивай.
Аноним 31/07/23 Пнд 08:44:28 759904 376
16907810481890.jpg 182Кб, 1000x706
1000x706
Почему юпитер и Сатурн такие красивые?
Аноним 31/07/23 Пнд 08:57:21 759906 377
>>759904
Это издали кажется. На самом деле это жуткие и обманчивые уёбы.
Аноним 31/07/23 Пнд 16:44:06 759921 378
Очередная новость про сверхпроводник комнатной температуры начала мелькать. Что скажете, опять дрисня, или есть шанс уже?
Аноним 31/07/23 Пнд 17:35:17 759927 379
>>759921
Ну пока никто в лабораторных условиях не повторил.
Аноним 31/07/23 Пнд 22:35:00 759940 380
>>759921
>Что скажете, опять дрисня, или есть шанс уже?
Перекроют какими-нибудь НЛО, революциями или японской катастрофой
Аноним 01/08/23 Втр 01:48:12 759952 381
>>759921
а смысл в комнатной температуре если там йоба-гигапаскали
Аноним 01/08/23 Втр 12:02:54 759959 382
>>759952
Ну для каких нибудь районных аккумов заряжающихся мгновенно без нагрева мегаваттами разве не пойдет? Это по сути и для каких-нибудь рейлганов подойдёт стационарных и на кораблях
Аноним 01/08/23 Втр 13:48:55 759964 383
>>759921
Очередная хуйня уровня бозона хиггса, ноль практических применений
Аноним 01/08/23 Втр 14:16:10 759966 384
А вот электрон испускает фотон с какого уровня? С того которого он спустился или на который он спустился?
Аноним 01/08/23 Втр 14:34:36 759967 385
>>759966
С которого спустился
Аноним 01/08/23 Втр 16:40:47 759969 386
>>759966
Фотоны и электроны "живут" в независимых от друг друга пространствах и уровни электрона нельзя привязать к фотону и наоборот.
Аноним 01/08/23 Втр 19:01:44 759970 387
>>759969
Хуйню пиздонул)) Как они могут жить в независимых пространствах, если они взаимодействуют? Вот ты дурак, блять))
Аноним 01/08/23 Втр 19:30:15 759972 388
>>759970
Ты так траллишь шолэ?
Собственно их взаимодействие происходит потому, что существуют(или определенны) в разных своих пространствах со своей "потенцией" или предпочтительностью существования. У этих пространств разная структура и геометрия. Электроны определены в спинорном пространстве, а фотоны в векторном. Само же взаимодействие это конкуренция "потенций".
Аноним 01/08/23 Втр 23:24:22 759983 389
О парадоксе Ферми.
Предположим, что на Земле разумная жизнь зародилась на миллиард лет раньше. Нет таких огромных запасов угля и нефти. НТП упирается в около предел. Не столкнется ли цивилизация с таким эффектом, как у нас на Земле с паровым двигателем, который изобрели хуй знает когда, а применили относительно недавно, когда производительность труда достигла мощности паровых движков. Ну то есть без огромных запасов нефти и угля, тупо не открыли бы ядерную энергетику.
Вот нашли инопланетные аборигены нефть, сожгли ее в керосиновых лампах. Был расцвет цивилизации, а потом топя китовым жиром через века, читали мифы о черном горючем из земли.
Быть может не главное, как расположена планета от звезды и вся эта хуйня. Главное момент зарождения цивилизация, что бы была энергия для развития цивилизации.
Без базового запаса энергии, ты физически не сможешь открыть новые источники энергии, будь то ветряки, солнечные панели, ГЭС, приливные электростанции, ядерную энергетику.
Аноним 01/08/23 Втр 23:45:26 759984 390
image 77Кб, 390x214
390x214
Не знаю куда написать. Буквально 5-10 мин. назад видел вот тут яркую вспышку, по яркости наверно сравнимую с Венерой. Как будто взрыв, секунда, две максимум.
Краснодарский край.

Что это могло быть? Не метеор точно, статичная была.
Аноним 01/08/23 Втр 23:51:10 759985 391
image 169Кб, 447x414
447x414
Аноним 02/08/23 Срд 02:10:43 759990 392
>>759984
А какого цвета вспышка была?
Аноним 02/08/23 Срд 09:32:21 759994 393
>>759983
>О парадоксе Ферми
Нету никакого "парадокса Ферми", забудь про эту соевую хуйню. Инопланетяне активно посещают Землю с древнейших времен
Аноним 02/08/23 Срд 09:44:18 759995 394
>>759984
Отражение от спутника или осколка на орбите?
Аноним 02/08/23 Срд 11:21:44 759998 395
image.png 471Кб, 1280x720
1280x720
>>757860
>Я читал, что наша Вселенная - пончки
Аноним 02/08/23 Срд 11:23:48 759999 396
Аноним 02/08/23 Срд 11:24:28 760000 397
image.png 50Кб, 200x200
200x200
Аноним 02/08/23 Срд 11:27:34 760001 398
>>759999
А ты докажи, что не сыр!
Аноним 02/08/23 Срд 11:31:00 760002 399
Аноним 02/08/23 Срд 12:04:17 760010 400
image.png 381Кб, 800x800
800x800
Аноним 02/08/23 Срд 13:13:49 760014 401
image 380Кб, 1159x824
1159x824
>>759990
Желто-оранжевая.

>>759995
Ты думаешь я не видел их? Слишком ярко и почти над горизонтом и с такой длительностью было бы видно смещение.

Чуть лучше сориентировался по месту в стеллариуме по Кассиопее, Пегасу и Капелле, помотал по времени +- 23.30, координаты г. Армавир. Ничего серьезного типо МКС там не пролетало.

тут последнее время Су-30 летают, но тоже не похоже, сообщений об авариях не было, помпаж вряд ли бы был так заметен, полигонов там рядом никаких нет, да и не слышал я чтобы летали в тот вечер
Аноним 02/08/23 Срд 13:21:27 760017 402
>>760014
>Слишком ярко и почти над горизонтом и с такой длительностью было бы видно смещение.
Алсо вчера в это время очень плохо было видно звезды, полнолуние + пыль, влажность, даже горизонт "подсвечивался" от этого.
Аноним 02/08/23 Срд 20:06:48 760050 403
>>759984
>>760014
Издалека прожектор на шасси пассажирного самолета выглядит как яркая звезда. У них они мощно лупят, очень-очень далеко видать. Может эта сушка так фонарем светила немного в твоем направлении и повернула?
Аноним 03/08/23 Чтв 01:09:02 760072 404
Анон, чем начальная сингулярность перед Большим Взрывом отличается от черной дыры? Почему пишут везде что в самом начале все вещество в сингулярности имело температуру Планковских значений, хотя должно быть как у черной дырки?
Аноним 03/08/23 Чтв 01:29:26 760073 405
>>760050
Нее, я жил рядом с аэродромом, я знаю о чем я пишу.
Причем тот район - это нихуя не район захода на посадку, не отработки ее.
Про пассажирские даже не стоит упоминать, там трассы закрыты с начала СBО, до этого много раз наблюдал самолеты и днем и ночью.

Я ж не просто так пишу, это было либо в ближнем космосе, либо в далеком космосе(вероятность крайне мала, но все же). Либо в атмосфере на большой высоте район бы я определил с дивана где-то тут "СЛОВО ИЗ СПАМ ЛИСТА" (село Красногвардейское на севере от Армавира и Ставрополя) и дальше с уходом на С-СВ

Скорее всего был болид, довольно крупный, хвост которого не был виден (выше писал про хуевую видимость), был виден только взрыв/разрушение под углом, от чего показался просто вспышкой, практически яркостью в Венеру при нормальной погоде.
Аноним 03/08/23 Чтв 03:53:06 760076 406
>>757854 (OP)

Знающие наблюдатели неба, как вы думаете, что на этом видео?

https://dropmefiles.com/AihRi

Прикрепить к сожалению не могу - не успевает загрузиться.
Аноним 03/08/23 Чтв 08:13:40 760088 407
>>760072
Сингулярность термин из математики и означает особенность, место где что-то ломается, обычно гладкость. В физике это плохая вещь и от их надо избавлять или как-либо чинить, в худшем случае скрывать.

Начальная сингулярность она же Космологическая сингулярность возникает тогда, когда мы прогнозируют эволюцию Вселенной в зад. В определенный момент все приходит к абсурду вроде вся Вселенная произошла из одного события из которого с бесконечной скоростью выпрыгнула. Внезапно Фиксится теорией Инфляции.

Сингулярность в черных дырах это уже особенность пространство-времени, при которой любая геодезическая внутри горизонта событий заводит в нее, что ломает всю физику, что сама ОТО становится не применима. Не фиксится, но есть надежды на квантовую гравитацию, где вместо сингулярность возникает что-то типа перехода пространства-времени в другое качество или вообще не возникает, вместо нее квантовая пена сычует на горизонте событий.
Аноним 03/08/23 Чтв 08:44:12 760089 408
>>760088
>Внезапно Фиксится теорией Инфляции
Никак она инфляцией не фиксится, просто отодвигается чуть дальше. Если ты попробуешь провести время дальше Планковской эпохи, то упрешься в ту же сингулярность - провести время дальше невозможно, т.к. его просто не существует, а уравнения ОТО выдают абсурдные бесконечные величины типа бесконечной плотности и бесконечной кривизны пространства-времени. Сингулярность вообще никак нельзя пофиксить. Надежды на квантовую гравитацию совершенно иллюзорны, поскольку неизвестно, когда ее построят и построят ли вообще: 60 лет уже пытаются строить и не получается абсолютно нихуя. Уверен, через 60 лет результат кардинально не поменяется
Аноним 03/08/23 Чтв 08:48:06 760090 409
>>760073
Да, может быть болид. Мы обычно их видим под углом, т.е. они прочерчивают линию, но по идее ты мог оказаться идеально на траектории так что тебе видна была только точка.
В принципе, так и могло бы выглядеть, почему бы и нет.
Аноним 03/08/23 Чтв 10:16:06 760111 410
>>760090
Да ты просто дебил
Аноним 03/08/23 Чтв 10:24:24 760113 411
Да не бомбит у [...].mp4 1233Кб, 1280x720, 00:00:07
1280x720
Аноним 03/08/23 Чтв 10:25:09 760114 412
>>760111
Разумеется, но какое это отношение к обсуждаемому вопросу?
Аноним 03/08/23 Чтв 10:28:08 760115 413
>>760114
Это антисетидебил, и у него горит жопа.
Не благодари.
Аноним 03/08/23 Чтв 10:42:19 760119 414
>>759969
>Фотоны и электроны "живут" в независимых от друг друга пространствах
В двух разных вселенных?
Аноним 03/08/23 Чтв 10:43:05 760120 415
>>759966
>А вот электрон испускает фотон с какого уровня?
А вот как фотон может испустить электрон - потому что у них одна природа
Аноним 03/08/23 Чтв 10:43:40 760121 416
Аноним 03/08/23 Чтв 10:44:50 760122 417
>>759904
>Почему юпитер и Сатурн такие красивые?
Потому что это не планеты в полном понимании - а прежде всего светилы/солнца, покрытые толстым слоем темных пятен.
Аноним 03/08/23 Чтв 10:46:33 760123 418
>>760115
Спасибо.
>Не благодари.
Ты мне не указ, понял, добрый человек‽
Аноним 03/08/23 Чтв 10:47:06 760124 419
>>759738
>Кто-нибудь знает что-то почитать про устройство бортового электрооборудования и програмное обеспечение ракет-носителей и космических аппаратов?
Это не тред про космос - это филиал раздела /sci
Аноним 03/08/23 Чтв 10:48:05 760125 420
>>759688
ну да - на лесках висят, а ты не знал?
Аноним 03/08/23 Чтв 10:49:08 760127 421
>>759673
влажность низкая
Аноним 03/08/23 Чтв 10:50:01 760128 422
>>759691
>за щеку тебе
детский сад
Аноним 03/08/23 Чтв 10:50:50 760129 423
>>759659
>Трещины, вызванные описанной выше деформацией, известны как усталостное разрушение
О, это машку надо
Аноним 03/08/23 Чтв 10:51:17 760130 424
>>759656
твоя мамка сам знаешь
Аноним 03/08/23 Чтв 10:51:40 760131 425
>>759655
И на Марсе вода есть
Аноним 03/08/23 Чтв 10:53:16 760132 426
>>759648
>Маск долбоеб эту хуйню начал с ядерками, теперь 99% думают что это способ терраформить планеты и создавать пригодные условия жизни на других планетах.
Маск вроде зеркала предлагал ещё, чем плохи зеркала, из пленки можно сделать зеркала? Из фольги?
Аноним 03/08/23 Чтв 10:53:52 760133 427
>>759642
>Блядь - это женщина
А женщина - мать твоя)
Аноним 03/08/23 Чтв 10:54:23 760134 428
>>759639
>С луной и так вся понятно
что пнятнот
Аноним 03/08/23 Чтв 10:55:31 760135 429
>>759626
>Планета не станет пригодной, т.к. луна не планета.
А Марс без общепланетарного магнитного поля - планета или прост большой камень?
Аноним 03/08/23 Чтв 10:56:37 760136 430
>>759561
>Вселенная уже с этими величинами рождается
Кто рождает Вселенную? Большой слон? Ты говно слона?
Аноним 03/08/23 Чтв 10:57:25 760137 431
>>759551
>тёмной материи и энергии не существует
А то мы блядь не знали без тебя
Аноним 03/08/23 Чтв 10:58:08 760138 432
Аноним 03/08/23 Чтв 11:16:52 760139 433
Аноним 03/08/23 Чтв 11:20:31 760141 434
>>760139
>Марс - планета
Без общепланетарного магнитного поля - не планета, а камень, как и Венера кстати
Аноним 03/08/23 Чтв 11:22:27 760142 435
Всё меньше Twitter, всё больше X: Илон Маск переименовал TweetDeck в XPro
В ближайшее время TweetDeck может стать полностью платным

ебанутый ваш машк
Аноним 03/08/23 Чтв 11:23:46 760143 436
F2YN81pXMAAjF1e.jpg 72Кб, 686x900
686x900
Космоновости PIR
>Как ожидается, в ближайшие дни Twitter (теперь уже X) сделает XPro/TweetDeck платной функцией.
Аноним 03/08/23 Чтв 11:25:27 760144 437
>Американский миллиардер Илон Маск обсудил с премьер-министром Монголии Лувсаннамсрайн Оюун-Эрдэнэ возможность создания учебных центров в пустыне Гоби для отправки людей на Марс.

Пустыня Гоби похожа на поверхность Марса, поэтому такой центр будет иметь большую эффективность.

Маск также добавил, что идея создания таких центров соответствует его целям по популяризации космических путешествий и колонизации Марса.
Аноним 03/08/23 Чтв 11:29:20 760145 438
>>760141
Я пользуюсь определением Международного Астрономического Союза, и тебе советую так же поступать, чтобы не было недопониманий.
Аноним 03/08/23 Чтв 11:36:29 760147 439
>>760145
Похуй на них - Марс это не планета!
Аноним 03/08/23 Чтв 11:37:48 760148 440
>>760147
Похуй на тебя, slaveянин.
Аноним 03/08/23 Чтв 11:44:43 760149 441
>>760147
>Похуй на них
Ну тогда с тобой не получится вести конструктивный диалог, если ты не будешь пользоваться общепринятой терминологией, это неизбежно приведет к недопониманиям и спорам.
Аноним 03/08/23 Чтв 11:50:02 760150 442
изображение.png 917Кб, 1024x587
1024x587
изображение.png 512Кб, 500x500
500x500
>>760149
>недопони
Исправляю.
Аноним 03/08/23 Чтв 11:51:50 760151 443
>>760119
Нет, привычная Вселенная это уже совокупность различных пространств или полей.
Аноним 03/08/23 Чтв 12:05:00 760153 444
i39.jpg 172Кб, 1920x1080
1920x1080
>>760149
>к недопониманиям и спорам
Ну пусть спорят? Камень без общепланетарного магнитного поля - не планета! А что другие пиздят - похуй!
Аноним 03/08/23 Чтв 12:07:57 760154 445
>>760150
Спасибо, хорошее исправление.

>>760153
Я думаю, что для того что ты описываешь, есть какое-то своё определение.
Аноним 03/08/23 Чтв 12:08:37 760155 446
145553541218829[...].jpg 114Кб, 803x994
803x994
>>760151
>привычная Вселенная это уже совокупность различных пространств или полей
Ну хуй знает какая тебе Вселенная привычна - ты тут оказывается пониеб, так что не удивлюсь, если ты пишешь про вселенную Marvel

Однако во Вселенной в которой мы обитаем - пространство одно, а иного и быть не может.
Аноним 03/08/23 Чтв 12:10:25 760156 447
>>760154
>есть какое-то своё определение
Нет - камень это не планета - или ты будешь спорить?
Общепланетарное магнитное поле - вот признак планеты, признак жизни. А Венера и Марс мертвы - это не планеты
Аноним 03/08/23 Чтв 12:12:18 760157 448
Российские системы РЭБ глушат Starlink ВСУ. Как тебе такое, Илон Маск?
Аноним 03/08/23 Чтв 12:24:23 760160 449
>>760156
>признак планеты
Теперь официально планетой Солнечной системы называется небесное тело, удовлетворяющее всем следующим условиям:

-обращающееся по орбите вокруг Солнца (то есть не являющееся спутником другого тела);
-обладающее достаточной гравитацией, чтобы иметь форму, близкую к шару;
-вблизи орбиты которого имеется «пространство, свободное от других тел».

Вот что подразумевается всеми под словом "планета". Ты задействуй новое слово для своего определения. "Жизнеспособная планета", может быть.
Аноним 03/08/23 Чтв 12:59:17 760163 450
>>760160
>вблизи орбиты которого имеется «пространство, свободное от других тел».
Ебать, а Земля-то по новой методичке планета? А то Луна всего в 4 раза меньше по радиусу. Практически такая же двойная планетная система, как Плутон с Хароном.
Аноним 03/08/23 Чтв 13:07:03 760165 451
>>760163
Планета. Барицентр внутри Земли. Через сколько-то лярдов лет, если не сгорят, когда Луну отнесет подальше и барицентр будет вне земли - можно будет считать двойной планетой.
Аноним 03/08/23 Чтв 13:37:33 760169 452
>>760088
Мой вопрос в том откуда разные источники берут у этой начальной сингулярности температуры порядка 10^32 и около того К? И почему вообще везде пишут эту цифру и не говорят почему вообще можно у этой сингулярности подходить с т. ч. феноменологической термодинамики и давать в ней определение температуры? Хотя, если совсем по-тупому сказать, мне по логике было бы понятней такая схема: много частиц в одном месте -> аддиктивно большая гравитация -> ооочень мало движения у частиц, ограничение их степеней свободы -> фактически черная дыра с околонулевой температурой.
Аноним 03/08/23 Чтв 14:16:08 760175 453
Почему Маск сказал, что нет никаких данных о инопланетянах, а он сильно в этой теме? Что с Тегеранским случаем, сливами НАСА о летающих на сверхскоростях по невозможным траекториям объектам? Ещё слышал про какие-то свидетельства кучи африканских детей и радиоактивные ожоги от взлёта тарелки у лингвиста, но нагуглить имя не смог.
Аноним 03/08/23 Чтв 15:12:28 760183 454
scale120092.jpg 126Кб, 500x560
500x560
>>760175
Так на самом деле нет никаких инопланетян!
Маску уж точно известно - он из рептилоидов
Летающие тарелки вылетают из полой Земли - там внутри развитая раса, которая не обнулялась катаклизмами на поверхности Земли
Так что то, что ты называешь инопланетянами - тоже по факту Земляне
Аноним 03/08/23 Чтв 15:13:50 760184 455
>>760163
>Ебать, а Земля-то по новой методичке планета? А то Луна всего в 4 раза меньше по радиусу. Практически такая же двойная планетная система, как Плутон с Хароном.
Земля планета - у Земли есть общепланетарное магнитное поле - а значит и внутреннее Солнце!
Аноним 03/08/23 Чтв 15:22:35 760186 456
>>760169
> откуда разные источники берут у этой начальной сингулярности температуры порядка 10^32 и около того К?
Искра Божья, по факту температура должна быть выше
Аноним 03/08/23 Чтв 17:22:16 760221 457
>>760169
Ты не задумывался как они вообще температуру считают? Ведь нет никаких измеряющих приборов, способных замерить её на таком расстоянии. Чисто выдают выдуманную инфу на выданные гранты, чтоб как-то отработать и получить новые, потому что космос на данный момент не изучаем, пока не придумают как безопасно летать хотя бы на скорости света.
Аноним 03/08/23 Чтв 17:38:04 760224 458
>>760221
>Ты не задумывался как они вообще температуру считают?
Ты, очевидно, только задумывался вместо того чтобы посмотреть на методологию, задумался такой, не придумал способа, и объявил ученых продажными шарлатанами.
Аноним 03/08/23 Чтв 18:11:44 760228 459
>>760224
Методология состоит из задумств и допущений. Никто ещё не летал дальше Луны, а уже вывалили кучу замеров всего и вся на миллионы световых лет.
Аноним 03/08/23 Чтв 18:22:16 760229 460
>>760136
>Кто рождает Вселенную?
Никто, она родилась сама собой
>>760169
>Мой вопрос в том откуда разные источники берут у этой начальной сингулярности температуры порядка 10^32
Ты путаешь сингулярность с Планковской эпохой. Уравнения ОТО дают абсурдные величины в условиях сингулярности - бесконечности, в т.ч. бесконечную температуру. Конечно, это не значит, что там действительно имели место бесконечности - природа их не терпит. Это лишь значит, что физика и математика в условиях сингулярности просто ломаются и перестают работать
>>760175
>Почему Маск сказал, что нет никаких данных о инопланетянах
Потому что он пиздабол на службе у жидовских глобалистских элит и всегда отрабатывает только нужную им повесточку
Аноним 03/08/23 Чтв 18:29:22 760230 461
>>760169
Большая температура(она кстати меньше чем платоновская) у Вселенной стала только после эпохи Инфляции, после того как инфлатронное поле выродилось и включились привычные нам поля. Тут и температуру и энтропию относительно легко оценить. До этого температура и энтропия были около нулевые.

Твоя схема не очень работает, потому что ОТОшная гравитация требует учета еще давления материи. Так до эпохи рекомбинации во Вселенной доминировало излучение и барионная материя не могла самогравитировать из-за высокого давления.
Аноним 03/08/23 Чтв 20:09:17 760232 462
>>760229
>она родилась сама собой
Ты хоть понимаешь насколько пизданутый бред несёшь?
Аноним 03/08/23 Чтв 22:40:46 760243 463
>>760228
Все наше восприятие мира
>состоит из задумств и допущений
Если по твоей логике играть, то можно до солипсизма доиграться.
Отрицать ты можешь сколько угодно, но вес твои слова будут иметь когда ты будешь разбираться в том, что отрицаешь.
Аноним 04/08/23 Птн 00:24:44 760248 464
>>760230
Спасибо, вроде понял. Можешь навскидку что нибудь посоветовать повкурить по теме? ОТО применительно к масштабам Вселенной. Может учебники для студиозусов каких, как вечернее чтиво. С выводами формул, историей.
Аноним 04/08/23 Птн 00:55:23 760249 465
>>760221
Задумывался. Это на самом деле несложно делать, озаботясь лишь вопросами как светит и откуда светит. Температура же - таки есть абстрактная величина, всегда косвенно определяется. Другое дело что за объект, например звёзды, которые можно рассматривать как квазистационарные системы, для которых можно даже дать определение равновесной температуры в наблюдаемый момент времени.
Ну вот про гранты - это по-любому могут выдумать что угодно.
Аноним 04/08/23 Птн 08:28:16 760263 466
>>760248
"Космология" и "Космология и гравитация" - Вайнберг.
Ну еще трехтомник "Гравитация" Торна, Минзера и Уилера.
По теории инфляции нет нормальной литературы, только научные статьи разной степенью цитируемости.
Аноним 04/08/23 Птн 08:35:38 760264 467
>>759985
Жираф золотым зубом сверкнул
Аноним 04/08/23 Птн 08:37:00 760265 468
>>760002
В вакууме его раздуло бы к хуям и он лопнул бы
Аноним 04/08/23 Птн 08:39:04 760266 469
>>758398
Распорки надо ставить, ато они просто ёбнутся друг на друга
Аноним 04/08/23 Птн 08:41:07 760267 470
>>760088
Сингуляторность вообще материальна?
Аноним 04/08/23 Птн 08:42:35 760268 471
>>760090
Он бы тогда анона захедшотил и он бы сюда не написал
Аноним 04/08/23 Птн 08:45:01 760269 472
>>759663
Ага, деньги за проезд брать, но ездить по нему не давать!
Аноним 04/08/23 Птн 08:50:03 760270 473
>>760268
Они полностью в верхних слоях сгорают.
Аноним 04/08/23 Птн 09:21:04 760271 474
Аноним 04/08/23 Птн 09:24:55 760272 475
>>760267
Сингулярность - это физико-математическое понятие, применимое к состоянию Вселенной в целом либо к локальной метрике пространства-времени (в случае черных дыр), когда собственно физика и математика перестают работать - все пространственно-временные координаты обращаются в ноль, а уравнения выдают абсурдные бесконечные величины
Аноним 04/08/23 Птн 10:06:46 760275 476
>>760272
Но внутри чёрной дыры должно ведь быть что-то материальное же.
Аноним 04/08/23 Птн 10:15:46 760276 477
>>760275
Внутри черной дыры сингулярность. Это значит, что наша физика и математика там не работают. Т.е. никто не знает, что находится в центре черной дыры и как оно работает. Можно лишь сказать, основываясь на том, что физика там идет по полнейшей пизде, - прежде всего, по пизде там идет само пространство-время, - что там заканчивается время как таковое, и любое тело, попавшее в центр ЧД, перестает существовать
Аноним 04/08/23 Птн 11:04:15 760278 478
>>760275
Необязательно материальное. Может быть энергия.
Аноним 04/08/23 Птн 11:27:25 760279 479
>>760278
Энергия по определению численное описание материи. Нематериальной энергии не может быть.
Аноним 04/08/23 Птн 11:56:46 760281 480
>>760275
Ясен хуй, что это "что-то материальное". Мы же в материальном мире живем. Просто эта форма материи не описывается в терминах пространства-времени, а описывается в каких-то иных
Аноним 04/08/23 Птн 12:01:04 760282 481
Аноним 04/08/23 Птн 13:24:41 760283 482
У массивных нейтронных звёзд есть и внутреннее ядро. Его радиус может достигать нескольких километров, плотность в центре ядра может превышать плотность атомных ядер в 10—15 раз.

Это как блять - плотнее атомных ядер? Нейтроны сжаты что ли?
Аноним 04/08/23 Птн 13:57:41 760284 483
>>760283
Средняя плотность вещества больше, чем в атомном ядре
Аноним 04/08/23 Птн 14:08:57 760285 484
Аноним 04/08/23 Птн 18:41:23 760293 485
>>760284
Больше, чем в ядре атома водорода?
Аноним 04/08/23 Птн 19:51:04 760295 486
image.png 6Кб, 549x557
549x557
>>760276
Вот чёрная точка - материальная сингуляторность, Я так понимаю? Пространство и время там есть, но в них нет смысла для тех кто далеко от чд.
Аноним 04/08/23 Птн 20:16:44 760299 487
Аноним 04/08/23 Птн 20:40:17 760302 488
>>760293
Да
>>760295
Там нет пространства-времени ни в каком смысле, поскольку все пространственно-временные координаты там обращаются в ноль. Сингулярность - это не точка в пространстве, а момент во времени. При пересечении горизонта сингулярность становится для тебя моментом будущего времени - про это часто говорят, что пространство и время в ЧД меняются местами. Достигнув же сингулярности, время для тебя закончится и ты перестанешь существовать
Аноним 04/08/23 Птн 22:31:10 760311 489
Пропускает ли аэрогель воду?
Пропускает ли аэрогель воздух?
Пропускает ли аэрогель радиацию?
Пропускает ли аэрогель жар?
Можно ли использовать аэрогель для создания орбитальных станций и даже ракет и всех составных частей ракеты-носителя от баков с топливом до сопл?
Аноним 04/08/23 Птн 22:38:50 760313 490
>>760302
>Достигнув же сингулярности, время для тебя закончится и ты перестанешь существовать

Это ведь бесконечная хуита, я прав? Сингулярность взрывается, образуется вселенная с галактиками. Потом звёзды коллапсируют, чёрные дыры с сингулярностями, сливаются в ещё больше чёрные дыры и вся вселенная в гравитационном капкане превращается в единую абсолютную сингулярность, которая снова взрывается вселенной и так до бесконечности.
Аноним 05/08/23 Суб 07:14:19 760334 491
Когда Млечный Путь упадет на Великий Аттрактор?
Аноним 05/08/23 Суб 09:54:02 760336 492
>>760313
>Это ведь бесконечная хуита, я прав?
Бесконечные величины получаются в уравнениях ОТО в применении к сингулярности. В природе бесконечностей не существует. Это лишь означает, что в условиях сингулярности известная нам физика не работает
>Сингулярность взрывается
Сингулярность - это не объект, и уж конечно это не какая-то гигантская бомба, которая "взрывается". Прежде всего это физико-математическое понятие, которое применяется к особенному случаю, когда функция обращается в ноль, а уравнения начинают выдавать абсурдные бесконечные величины, т.е. в данном особенном случае перестают адекватно работать.
Космологическая сингулярность - это момент в прошлом Вселенной, дальше которого время проложить невозможно - оно на этом моменте заканчивается; в применении к нему все пространственно-временные координаты обращаются в ноль, а уравнения ОТО дают бесконечные ответы. Т.е., еще раз, это не объект, не какая-то "точка", из которой "все вылетело". Это особенное состояние, в котором находилась вся Вселенная и которое современная физика не умеет описывать.
Гравитационная сингулярность в черных дырах - это также не объект, не "точка", а момент в будущем для наблюдателя, который пересек горизонт. (Именно поэтому покинуть черную дыру невозможно - для этого потребовалась бы машина времени.) Как и в случае с космологической сингулярностью, здесь все пространство-время обращается в ноль, уравнения начинает пидарасить и выводить на бесконечности, а наблюдатель, который достигает момента сингулярности, заканчивает свое существование
>сливаются в ещё больше чёрные дыры и вся вселенная в гравитационном капкане превращается
Не все ЧД сливаются друг с другом. К тому же, Вселенная расширяется с ускорением, и вместе с этим расширением разносятся друг от друга и черные дыры. Кроме того, черные дыры не вечные - они со временем испаряются. Поэтому в будущем Вселенную ждет не коллапс обратно в сингулярность, а вечное ускоренное расширение и тепловая смерть
Аноним 05/08/23 Суб 09:55:17 760337 493
>>760334
Никогда. Великий Аттрактор удаляется от нас в результате расширения Вселенной, так же как и мы от него
Аноним 05/08/23 Суб 11:39:53 760341 494
>>760337
>Никогда
Глупость)) Вселенная сейчас расширяется, но потом начнется сужаться. Не?
Аноним 05/08/23 Суб 12:00:19 760342 495
>>760341
>Не?
Нет. Вселенная будет вечно расширяться с ускорением, что обусловлено доминированием темной энергии
Аноним 05/08/23 Суб 12:16:14 760345 496
>>760299
Чо, никто трансляции не смотрит?
Аноним 05/08/23 Суб 13:06:23 760348 497
>>760342
Схуяли? Мы про тёмную энергию ничего не знаем. Венециано в интервью недавно сказал, что возможны любые варианты. Ускоренное расширение началось по малопонятным причинам всего 6 миллиардов лет назад, и по таким же причинам может и закончиться.
Аноним 05/08/23 Суб 13:21:39 760349 498
>>760342
Темная чё? Типо Дарт Вейдер? Это ж глупость, не?
Аноним 05/08/23 Суб 13:44:27 760351 499
>>760348
>Схуяли?
Последние наблюдательные данные согласуются с вакуумной моделью темной энергии. Темная энергия неотличима от вакуума с ненулевой плотностью. Если в дальнейшем появятся данные, противоречащие этой модели, но говорящие в пользу, скажем, квинтэссенции или, что менее вероятно, в пользу фантома, тогда и можно будет говорить об иной судьбе Вселенной. А сейчас положняк такой
Аноним 05/08/23 Суб 19:00:54 760360 500
Если подлететь к чорной дыре, скинуть с пепелаца супер прочный трос а на нем специально обученного чювака и окунуть его под горизонт на пол шишки, его можно будет достать целым? Я понимаю спагетти вся хуйня, но допустим дырка оче массивная и разница гравитации не порвет очко чюваку.
Аноним 05/08/23 Суб 19:31:36 760363 501
>>760360
Молекулярные связи на горизонте событий не будут работать, ты достанешь обрубок. У тебя трос из молекул, а молекулы держатся за счет обмена фотонами между электронными облаками. Фотоны из-за горизонта событий не будут долетать до атомов снаружи, связь нарушена, целостность объекта потеряна.
Аноним 05/08/23 Суб 19:58:50 760366 502
>>760363
Но ведь насколько я понимаю под горизонт сверхмасивной можно залезть и даже целым но потом один хуй разбиться об сингулярность. Так че за магия там рвет молекулы именно на этой границе? Или под горизонт все попадает в виде каши из частиц уже, даже без спагетти? Типа трос будет проходить через горизонт как через мясорубку?
Аноним 05/08/23 Суб 20:01:24 760367 503
>>760363
Или например удерживать объект не тросом, а супер мощным магнитом? Магнитное поле вроде не из молекул хуй знает.
Аноним 05/08/23 Суб 20:03:27 760368 504
>>760366
Я же сказал - на ГС у тебя сигнал обратно не доходит. Вот определенная область, которая именуется горизонтом событий черной дыры, она характеризуется тем, что никакие взаимодействия (по крайней мере фотоны - точно) назад не вылезают.
Вот на этой границе то что залезло - оно не коммуницирует с тем что снаружи, оно не может, нечем.
>>760367
Это то же электромагнитное взаимодействие что и удерживает атомы в молекулах и не позволяет тебе рассыпаться в микроскопическую кучку ионов и электронов.
Аноним 05/08/23 Суб 20:28:05 760370 505
>>760368
А можно выйти на орбиту под горизонтом вокруг сингулярности? Вроде тогда не надо будет его пересекать изнутри значит скорость нужна будет чуть меньше световой и теорически это возможно вообще или нет?
Аноним 05/08/23 Суб 20:55:43 760373 506
Че за тема с альфой бетой и проксимой Центавра? Это двойная или тройная система? Там вообще как норм? Можно ли рассматривать как альтернативу солнечной системе?
Аноним 05/08/23 Суб 21:29:25 760374 507
Есть тупой вопрос, кстати. Чому не перекатываете?
Аноним 05/08/23 Суб 21:34:58 760375 508
>>760370
Поговаривают что да. На деле хуй знает как там дела обстоят, там же метрика повернута.
Аноним 05/08/23 Суб 21:35:16 760376 509
>>760374
А зачем перекатывать?
Аноним 05/08/23 Суб 23:14:40 760382 510
>>760351
положняк такой — в говне мочёные хотят кушать.
Аноним 06/08/23 Вск 08:29:21 760387 511
Ответить в тред Ответить в тред

Check this out!

Настройки X
Ответить в тред X
15000
Добавить файл/ctrl-v
Стикеры X
Избранное / Топ тредов